You are on page 1of 85

TEST 1 ANSWERS

Test 1
1. In which of the following congenital anomalies of the facial region the newborns have difficulties in eating?
a) Cleft palate
b) Coloboma
c) Microstomia

2. What is the most appropriate age for having a reconstructive surgical procedure of a cleft lip?
a) The first 6 months after birth
b) After the p t year of the baby
c) In puberty

3. What is the reason for developing cleft palate anomaly?


a) Birth trauma
b) Disorders in embryogenesis
c) Complications of facial tumors

4. What is the major and definitive treatment for congenital anomalies of the facial region?
a) Surgical
b) Conservative

5. Which are the bronchiectasis types?


a) Congenital
b) Acquired
c) Hereditary

6. Which of the following statements is true for spontaneous pneumothorax?


a) Sudden beginning
b) There is a pathological communication between the lung and the pleural cavity
c) Lung parenchyma is intact
d) All of the above

7. What is the reason for perforation of emphysematous bullae?


a) Thin walls of the bullae
b) High intraalveolar pressure
c) High intrathoracic pressure

8. What is the clinical manifestation of spontaneous pneumothorax?


a) Pain
b) Cough
c) Dyspnea
d) Haemoptysis

9. Which of the following surgical procedures includes anastomosis between the stomach and the jejunum
(gastrojejunal anastomosis)?
a) Meckel diverticulum resection
b) Total gastrectomy

1
TEST 1 ANSWERS

c) Heineke-Mikulicz pyloroplasty
d) Billroth 1 resection
e) Billroth 2 resection

10. What is the position of the right vagus nerve in the distal part of the esophagus?
a) In front of the esophagus
b) Behind the esophagus
c) Left from the esophagus
d) Right from the esophagus

11. Which of the following are ulcer disease complications?


a) Perforation
b) Bleeding
c) Thrombosis
d) Penetration
e) Malignant transformation
f) Stomach dilation
g) Pyloric stenosis
h) Pyloric dilation

12. Which of the following inguinal hernia repair techniques includes continuous suture?
a) Bassini technique
b) Shouldice technique
c) Postemsky technique
d) McVay technique

13. What is the Cooper test purpose?


a) To distinguish indirect from direct type of inguinal hernia
b) To distinguish indirect type of inguinal hernia from femoral hernia
c) All of the above
d) Non of the above

14. Is it possible for the groin hernia sac to reach the scrotum?
a) It never reaches the scrotum
b) It always reaches the scrotum
c) It sometimes reaches the scrotum
d) Non of the

15. What is Whipple procedure?


a) Local tumor excision
b) Total pancreatectomy
c) Distal pancreatectomy
d) Proximal duodenopancreatic resection

16. How could bile duct derivation be performed in patients with jaundice due to pancreatic head ca rcinoma?
a) Stent insertion in the common bile duct

2
TEST 1 ANSWERS

b) Cholecystogastric anastomosis
c) Choledochojejunal anastomosis
d) All of the above

17. Which of the following statements a bout pancreatic cancer is wrong?


a) Contrast CT-scan is a diagnostic method of choice
b) Abdominal ultrasound examination is a diagnostic method of choice
c) Resectability assessment of the tumor is made during the operation
d) Portal vein infiltration is a total contraindication for radical surgical treatment
e) Pancreatic carcinoma is sensitive to radio- and chemotherapy

18. Which parts of the GIT are affected in Crohn's disease?


a) Ileum
b) Colon
c) The whole GIT- from the mouth to the anus

19. Which sex is more affected in Crohn's disease?


a) Female
b) Male
c) Incidence is equal in both sexes

20. What is the etiology of Crohn's disease?


a) Viral infection
b) Mesenteric blood supply disorders
c) Allergy
d) Unclear etiology
e) All of the above

21. What is the clinical manifestation of Crohn's disease?


a) Alternation of constipation and diarrhea
b) Rectorrhagia, subfebrility
c) Abdominal pain

22. Which age group is most commonly affected in intestinal intussusception?


a) 0 - 6-7 months
b) 10 - 20 years
c) 40 - 50 years
d) 65 - 75 years
e) All of the above

23. Which is the most common localization of intestinal intussusception?


a) Small intestines
b) Sigmoid colon
c) Rectosigmoid segment
d) lleocecal segment
e) Transverse colon

3
TEST 1 ANSWERS

24. What is the possible diagnosis of a restless 6-month-old baby with palpable mobile tumor mass in the right part
of the abdomen?
a) Testicle torsion
b) Intestinal intussusception
c) Allergy
d) The baby is hungry
e) Non of the above

25. What are the medical terms for the intussusceptions parts?
a) lntussusceptum and intussuscipiens
b) Proxima I and distal segment
c) Ileum and cecum
d) All of the above
e) None of the above

26. Which are the 4 major arteries that supply the stomach?
a) Left gastric artery
b) Common hepatic artery
c) Right gastroepiploic artery
d) Left gastroepiploic artery
e) Superior mesenteric artery above

27. Which of the following statements a bout pancreatic fistulas is wrong?


a) They form after a traumatic injury or inflammation of the pancreatic tissue
b) The secretion contains amylase which exceeds many times plasma levels
c) The amount of secretion could be more than 1000ml
d) Somatostatin/Sandostatin reduces pancreatic secretion and helps the recovery
e) Chronic secretion doesn't lead to healing and has a higher mortality rate

28. Which of the following techniques is best for simultaneous treatment of both inguinal and femoral hernia?
a) Bassini technique
b) Martinov technique
c) Postemsky technique
d) McVay technique

4
Test 2 Answers

Test 2
1. What is the surgical treatment of the cleft lip?
a) Uraloplasty
b) Rhinoplasty
c) Cheiloplasty

2. In which of the following traumatic injuries of the facial region there is a haematoma in the shape of sunglasses?
a) Maxillar fracture
b) Gunshot injury
c) Puncture injury

3. Which is the most common complication in parotid gland injury?


a) Fistula
b) Haemorrhage
c) Paralysis of the facial muscles

4. What happens after traumatic injury of the facial nerve?


a) Paralysis of the facial muscles
b) Dysphagia
c) Motor aphasia

5. How spontaneous pneumothorax could be diagnosed?


a) By patient's history
b) By percussion
c) By auscultation
d) By X-ray
e) By thoracoscopy
f) All of the a hove

6. What does treatment of spontaneous pneumothorax include?


a) Antibiotic infusions
b) Single pleural puncture
c) Thoracocentesis
d) Thoracotomy

7. Which of the following statements is false for pleural empyema ?


a) There is an inflammatory process in the pleural space
b) There is a bacterial contamination of the pleural cavity
c) There is an air inside the pleural cavity
d) There are thick adhesions between the parietal and visceral layer of the pleura

8. How many types of pleural empyema are there?


a) Total
b) Apical

1
Test 2 Answers

c) Basal
d) Mediastinal
e)AII of the above

9. Chronic hypertrophic gastritis is the disease of:


a) Zollinger-Ellison
b) Menetrier
c) Peutz-Jeghers
d) Forrest
f) Heineke-Mikulicz

10. Which is the most common classification of upper GIT bleeding?


a) Zollinger-Ellison
b) Menetrier
c) Peutz-Jeghers
d) Forrest
e) Heinike-Mikulicz

11. Which stomach layers are affected in early stomach cancer?


a) Mucosa
b) Mucosa and submucosa
c) Mucosa, submucosa, muscular layer
d) Mucosa, submucosa, muscular layer, serosa

12. Which is the most common histological type of stomach carcinoma?


a) Squamous cell carcinoma
b) Adenosquamous carcinoma
c) Adenocarcinoma
d) Non differentiated
e) Neuroendocrine tumor

13. What is the definition of hernia taxis?


a) Type of surgical procedure
b) Manual reduction of strangulated hernia
c) All of the above
d) Non of the above

14. What is the first method of choice in a baby with incarcerated inguinal hernia?
a) Hernia taxis
b) Urgent surgical treatment
c) Clinical observation
d) All of the above
e) Non of the above

2
Test 2 Answers

15. In which cases surgical treatment of inguinal hernia is necessary in babies under 8 months?
a) Never
b) Always
c) If there are frequent incarcerations
d) All of the above
e) Non of the above

16. ls it possible to have a prevascular position of the hernia sac in a femoral hernia?
a) Never
b) Sometimes
c) Always

17. What is splenosis?


a) Caudate spleen
b) Big spleen
c) Ectopic spleen
d) Accessory spleen
e) Spleen with blood vessels abnormalities

18. What is the most dangerous complication of wandering spleen?


a) Infection and sepsis
b) Thoracic localization and dyspnea
c) Splenic torsion
d) Spontaneous rupture
e) Compression of the stomach and pancreas

19. What is the most common cause of spleen infarction?


a) Congenital ano malies of the spleen
b) Migrating embol from the left atrium of the heart
c) Pulmonary thromboembolism
d) Thalassemia major
e) Essential thrombocytopenia

20. Which are the typical clinical features of Crohn's disease?


a) Alternation of constipation and diarrhea
b) Abdominal pain, diarrhea, loss of weight
c) Loss of weight, anemia
d) Abdominal pain and disten sion, constipation, vomiting

21. Which layers of the intestinal wall are affected in Crohn' s disease?
a) Serosa
b) Mucosa and se rosa
c) All the layers
d) Only mucosa

3
Test 2 Answers

22. What are the macroscopic changes of the intestinal wall in Crohn's disease?
a) Thick walls, narrowed lumen
b) Thin walls, widened lumen
c) There are no intestinal wall changes

23. Which are the complications of Crohn's disease?


a) Anal abscess and fistula formation
b) Intestinal fistulas
c) There are no complications

24. Which of the following arterial blood vessels is most commonly affected by embolism as a mesenteric blood
flow disorder?
a) Superior mesenteric artery
b) Inferior mesenteric artery
c) Celiac trunk
d) Iliac artery

25. Which of the following diagnostic methods is useful in mesenteric thrombosis?


a) CT angiography
b) Doppler ultrasound
c) Seldinger technique catheter angiography
d) All of the above

26. Which are the risk factors for arterial mesenteric thrombosis?
a) Atherosclerosis
b) Atrial fibrillation
c) Acute pancreatitis
d) Bronchial asthma
e) Acute intestinal obstruction

27. The arc of Riolan connects branches of:


a) Superior mesenteric vein and inferior mesenteric vein
b) Internal iliac artery and external iliac artery
c) Superior mesenteric artery and inferior mesenteric artery
d) Common iliac artery and superior mesenteric artery

28. Which of the following are endocrine tumors of the pancreas?


a) Thymoma
b) Pheochromocytoma
c) lnsulinoma
d) Glucagonoma
e) Gastrinoma

4
Test 2 Answers

5
Test3 Answers

Test 3

1. What are the major characteristics of a gunshot injury?


a) Forming a wound canal
b) Presence of a projectile in the affected tissues
c) Loss of sight
d) Forming a haematoma

2. Which of the following factors should be taken into consideration when forming a diagnostic algorithm for
closed traumas of the facial region?
a) Psychological condition of the patient
b) Type and localization of the traumatic agent, general condition of the patient, type of surgical approach,
extent of the traumatic injury
c) Trauma circumstances

3. Which of the following neoplasms is malignant and has a metastatic potential?


a) Melanoma
b) Basocellular carcinoma
c) Atheroma

4. Which of the following is a complication of fur uncle?


a) Abscess formation
b) Spontaneous drainage of the furuncle content
c) Venous vessels transmission to the cavernous sinus

5. Which of the following statements is true for treatment of pleural empyema?


a) It requires evacuation of the pus
b) It requires quick therapeutical measures for the inflammatory process
c) It aims lung expansion
d) Pleural puncture is a method of choice

6. What is mediastinitis?
a) Pleural inflammation
b) lnflammaton of surrounding tissues to the mediastinum
c) Inflammation of the mediastinal tissues

7. What is the medical term for a pus collection in the pleural space?
a) Chylothorax
b) Hydrothorax
c) Haemothorax
d) Pyothorax

8. What is the medical term for a lymph collection in the pleural space?
a) Chylothorax
b) Pneumothorax

1
Test3 Answers

c) Pyothorax
d) Hydrothorax

9. What is Laugier hernia?


a) Femoral hernia at a typical position
b) Prevascular type of femoral hernia
c) Femoral hernia passing through the lacunar ligament
d) All of the above
e) Non of the above

10. Is it possible the hernia sac in femoral hernia to pass through the muscular lacuna?
a) Never
b) Sometimes
c) Always

11. Where is the exact position of the intumescence in femoral hernia?


a) Below the inguinal ligament
b) Above the inguinal ligament
c) All of the above
d) Non of the above

12. ls it possible that men could have inguinal and femoral hernia at the same time?
a) Never
b) Rarely
c) Always

13. Which of the following surgical procedures are radical treatment for stomach cancer?
a) Tumor debulking
b) Subtotal gastrectomy
c) By-pass anastomosis
d) Total gastrectomy
e) Hepatectomy

14. In which cases stomach carcinoma is indicated for adjuvant chemotherapy?


a) In all patients
b) Depends on the histological type and the TNM stage
c) In young patients
d) In old patients
e) Depends on the localization of the tumor

15. According to the incidence of stomach cancer Bulgaria is one of the countries with:
a) Highest morbidity
b) High morbidity
c) Medium morbidity
d) Low mo rbidity

2
Test3 Answers

16. Which of the following statements a bout acute cholecystitis is false?


a) Nausea, vomiting
b) Positive Kerh sign
c) Positive Murphy sing
d) Muscular defense in the right hypochondriac area and positive Blumberg sign
e) Positive Myussi-Georgievsky

17. Splenectomy in hereditary spherocytosis is indicated in which of the following age groups?
a) In newborns
b) In teenagers
c) 30-40-year-old
d) In mature age
e) Splenectomy is contraindicated

18. Splenectomy in thalassemkia minor is indicated in which of the following age groups?
a) In newborns
b) In teenagers
c) 30-40-year-old
d) In mature age
e) Splenectomy is contraindicated

19. Which of the blood cells are destroyed in the spleen in patients with Werlhof disease?
a) Erythrocytes
b) Leucocytes
c) Platelets
d) Lymphocytes
e) All of the above

20. Which of the following cells could be detected in patients with Hodgkin's disease after splenectomy?
a) Psammoma bodies
b) Sternberg cells
c) Hill cells
d) Foreign body cells
e) Defect erythrocytes

21. What is the differential diagnosis of Crohn's disease?


a) Chronic ulcerous colitis
b) Acute appendicitis
c) Diverticulosis
d) Intestinal obstruction
e) All of the above

22. Which diagnostic methods could be used in Crohn's disease?


a) Abdominal ultrasound examination

3
Test3 Answers

b) Patient's history and laparoscopy


c) Patient's history, colonoscopy with biopsy, contrast X-ray
d) lrigography

23. What is the treatment of Crohn's disease?


a) Conservative
b) Surgical
c) Conservative followed by surgical if there are complications

24. What does surgical treatment of Crohn's disease include?


a) lleostomy
b) Segmental resection
c) Colostomy
d) Colectomy
e) Biopsy and conservative treatment

25. Which parts of the intestines are affected in massive thrombosis of the superior mesenteric artery?
a) Duodenum
b) Jejunum and ileum
c) Cecum and ascending colon
d) Descending colon and sigmoid colon
e) Rectum

26. What is the normal amount of fluid in the abdominal cavity?


a) 20ml
b) 80ml
c) 120ml
d) 200ml
e) 220ml

27. Which of the following diseases does not lead to forming a hemorrhagic exudate in the abdominal cavity?
a) Acute pancreatitis
b) Acute cholecystitis
c) M esenteric thrombosis
d) Mesenteric embolism
e) Acute intestinal obstruction

28. Abdominal pain as an initial sympto m in acute peritonitis is present in:


a) 50% of the patients
b) 50 - 75 % o f the patients
c) 75 - 90 % of the patients
d) 100 % of the patients

4
Test4 Answers

Test4

1. What is the treatment of facial erysipelas?


a) Surgical
b) Antibiotic

2. Which is the microbiological agent that causes erysipelas?


a) Beta haemolytic streptococcus
b) E.coli
c) Pseudomonas aeruginosa

3. Is there a possibility for developing distant metastasis in melanoma?


a) Yes, the tumor has a metastatic potential
b) No, the tumor has only one localization

4. What is the origin of the mid line cysts of the neck?


a) Thyroglossal duct*
b) Thymus gland
c) lnfrahyoid muscles

5. What is the medical term for a blood collection in the mediastinum?


a) Mediastinitis
b) Haemomediastinum
c) Pneumomediastinum
d) Haemothorax

6. What is the definition of pneumomediastinum?


a) A blood collection in the mediastinum
b) An air collection in the mediastinum
c) A pus collection in the mediastinum
d) A lymph collection in the mediastinum

7. What is myasthenia gravis?


a) Neuromuscular disorder causing muscle weakness due to absence of acetylcholine receptors in the
postsynaptic membrane
b) Electrolyte and enzyme transport disorder leading to muscle weakness
c) Neuromuscular disorder due to nutrient insufficiency

8. What is the clinical manifestation ofvena cava superior syndrome?


a) Oedema of the head, neck, upper extremities and the upper part of the thorax
b) Oedema of the chest and the abdomen
c) Oedema of the head, the chest and the abdomen
d) Oedema of the head, neck, upper and lower extremities

9. Cholecystostomy is indicated in acute destructive cho lecystitis in:

1
Test4 Answers

a) Concomitant acute edematous pancreatitis


b) Concomitant obstructive jaundice
c) Poor general condition of the patient
d) Concomitant chola ngitis
e) All of the above

10. What possible complications of the major disease have developed in a patient with acute phlegmonous
cholecystitis, fever to 38, obstructive jaundice, no signs of acute peritonitis?
a) Stenosis of the papilla
b) Gallbladder empyema
c) Pylephlebitis
d) Subhepatic abscess
e) Purulent cholangitis

11. Which of the following is an indication for an emergency surgery in acute cholecystitis?
a) Intensity of the pain
b) Duration of the disease
c) Number of colic pain episodes in the past
d) Presence of acute peritonitis
e) Presence of stones

12. Which of the following cases is indicated for retrograde cholecystectomy?


a) In old patients
b) If there are signs of acute cholangitis
c) If there is a gallbladder contraction
d) If there is an incarcerated gallstone in the gallbladder neck
e) If there are inflammatory changes of the gallbladder neck

13. Is it common for the femoral hernia to incarcerate?


a) Never
b) Sometimes
c) Often

14. Could femoral hernia become big in size?


a) Often
b) Never
c) Rarely

15. Which is the most common type of incarceration in femoral hernia?


a) Richter type
b) Litre type
c) All of the above
d) Non of the above

16. Could hernia taxis be performed in femoral hernia?

2
Test4 Answers

a) Always
b) Sometimes
c) Never

17. In which of the following conditions splenectomy is indicated?


a) Spleen abscess
b) Thalassemia minor
c) Splenic vein thrombosis
d) Splenitis
e) Wandering spleen

18. Which anatomical feature could lead to a dangerous complication after splenectomy?
a) Spleen position
b) Spleen haematoma during splenectomy
c) Short gastric arteries arising from the splenic artery
d) Enlargement of the spleen
e) Spleen's ligaments

19. Which of the following is a contraindication for splenectomy?


a) High levels of platelets
b) Low levels of leucocytes
c) Low levels of neutrophils
d) High level of hemoglobin
e) Low level of platelets

20. Which of the following spleen tumors are benign?


a) Splenoma
b) Hemangioma
c) Fibrosa rcoma
d) Angiosarcoma
e) Lymphangioma

21. Which of the following etiology factors are typical for diverticulosis?
a) Congenital tissue w eakness
b) High intraluminal pressure
c) Discoordination in peristalsis
d) Hereditary risk factors
e) All of the above

22. Which of the following diagnostic me thods could not be used in colon diverticu losis?
a) Colonoscopy
b) CT-scan
c) Upper endoscopy
d) lrigography

3
Test4 Answers

23. What does the conservative treatment of acute diverticulitis include?


a) Antibiotic therapy and dietary regime
b) Non steroid anti-inflammatory drugs
c) 5-ASA medications
d) Spasmolytics

24. What is megacolon?


a) Congenital/acquired disease of the colon which leads to dilation and enlargement of the colon and motility
disorders
b) Congenital disease of the small intestines which leads to their dilation and enlargement,there are motility
disorders\congenital/acquired disease of the colon and diarrhea

25. What is the therapeutic approach to a 14-year-old girl diagnosed with pneumococcal peritonitis who suffered
from acute bronchitis for 3 days and after that developed diffuse abdominal pain, nausea, vomiting, diarrhea,
circumoral cyanosis, HR 100 bpm, abdominal rigidity, leucocytosis to 38 000, elevated ESR level?
a) Emergency surgery
b) Antibiotic therapy
c) Detoxication therapy

26. 46-year-old patient is operated for acute gangrenous appendicitis and develops dull pain around the umbilicus,
rectal tenesmus and increased body temperature to 37.8 - 38.5 5 days after surgery. What is the diagnosis?
a) Acute proctitis
b) Inflamed hemorrhoids
c) Diffuse peritonitis
d) Douglas pouch abscess
e) Acute cystitis

27. What is the most common cause for acute peritonitis?


a) Acute appendicitis
b) Perforated ulcer
c) Acute salpingitis
d) Small intestine strangulation
e) Stomach carcinoma

28. What is the duration of the reactive stage of acute peritonitis?


a) 4 - 6h
b) 24h
c) 48h
d)72h
e) M o re than 72h

4
Test 5 Answers

Test 5

1. What is the cause of torticollis (wry neck)?


a) Birth trauma
b) Embryogenesis disorders
c) Disorders of the neck muscles innervations

2. What is the medical term for neck lymphadenitis in TB?


a) Tuberculum
b) Scrofulosis
c)Lipomatosis

3. Which of the following neck tumors is benign?


a) Haemangioma
b) Lymphosarcoma
c) Hodgkin lymphoma

4. Which of the following conditions could develop in neck veins injury?


a) Air embolism
b) Haemorrhage shock
c) Infection

5. How many types of esophageal atresia are there depending on the communication between the esophagus and
the trachea?
a) 3 types
b) 4 types
c) 5 types

6. Esophageal perforation due to chemical burns of the esophagus requires:


a) Urgent surgical treatment of the mediastinitis
b) Prolonged clinical observation, surgical treatment in case of worsening
c) Elective surgical procedure after an informed consent of the patient

7. In which of the following conditions upper endoscopy is contraindicated?


a) Esophageal carcinoma
b) Esophageal perforation
c) Haemathemesis
d) Lung carcinoma

8. Which are the anatomical narrowings of the esophagus?


a) Thoracic and abdominal
b) Upper, middle and lower
c) Cervical, diaphragmatic and bowel

9. Which is the first diagnostic method of choice in 81-year-o ld patient with phlegmonous cholecystit is?

1
Test 5 Answers

a) Abdominal ultrasound examination


b) Cholecystocholangiography
c) Laparoscopy
d) Retrograde cholangiopancreatography
e) Percutaneous transhepatic cholangiography

10. What kind of surgical procedure must be performed in 81-year-old patient with acute phlegmonous
cholecystitis in a poor general condition?
a) Cholecystectomy
b) Cholecystostomy
c) Laparoscopic drainage of the subhepatic space
d) Cholecystolithotomy

11. What kind of procedure must be performed in a non complicated cholelithiasis which is an accidental finding?
a) Endoscopic retrograde cholangiopancreatography
b) Laparoscopy
c) Abdominal ultrasound examination
d) Percutaneous transhepatic cholangiography

12. Which of the following is not a complication of acute cholecystitis?


a) Obstructive jaundice
b) Portal hypertension
c) Purulent cholangitis
d) Subhepatic abscess
e) Incarcerated gallstone in the papilla

13. Which of the following conditions must be included in the differential diagnosis of femoral hernia?
a) Inguinal hernia
b) Incarcerated inguinal hernia
c) All of the above
d) Non of the above

14. Which of the following conditions must be included in the differential diagnosis of femoral hernia?
a) Great saphenous vein varicosis
b) Lymphadenitis
c) All of the above
d) Non of the above

15. Which of the following conditions must be included in the differential diagnosis of femoral hernia?
a) Lymphosarcoma
b) Femoral artery aneurysm
c) All of the above
d) Non of the above

16. What is the treatment of femoral hernia?

2
Test 5 Answers

a) Surgical
b) Conservative
c) Non of the above

17. Which of the blood cells are destroyed in the spleen in patients with Felty syndrome?
a) Erythrocytes
b) Leucocytes
c) Platelets
d) Lymphocytes
e) All of the above

18. Which are the typical features of caudate spleen?


a) Enlargement of the spleen and compression of the stomach
b) Spleen is thin and long, could reach the pelvis
c) Could be located in the mesenterium or the great omentum
d) Could form after a traumatic injury of the spleen
e) It is commonly located in the right part of the abdominal cavity

19. Which are the major diagnostic methods for spleen echinococcosis?
a) Abdominal CT-scan
b) Abdominal ultrasound examination
c) Spleen scintigraphy
d) Abdominal X-ray
e) Laparoscopy and spleen biopsy
f) Upper endoscopy

20. What is spleen tuberculosis?


a) Acute nonspecific infection
b) Fulminant infection
c) Acute specific infection
d) Chronic specific infection
e) Chronic nonspecific infection

21. How many etiopathogenetic types of megacolon are there?


a) Congenital
b) ldiopatic
c) Symptomatic
d) All of the above

22. What is the incidence of Hirschhprung's disease?


a) 1:5 000
b) 1:10
c) 1:1 000 000

23. What is the clinical manifestation of Hirschprung's disease?

3
Test 5 Answers

a) Loss of appetite, nausea, vomiting, constipation


b) Recurrent diarrhea, fever, abdominal pain
c) Constipation right after birth, defecation in 3-5 days after enemas

24. How many forms of Hirschprung's disease are there?


a) Compensated
b) Subcompensated
c) Decompensated
d) All of the above

25. What is the infection's spreading way in primary peritonitis?


a) From stomach ulcer perforation
b) From appendiceal perforation
c) From acute adnexitis
d) Hematogenous way
e) From intestinal trauma

26. Which of the following symptoms is not typical for acute peritonitis?
a) Muscular defense
b) Courvoisier sign
c) Tachycardia
d) Stop offlatulence
e) Vomiting

27. Which of the following symptoms is not typical for acute peritonitis?
a) Tachycardia
b) Dry tongue
c) Muscular defense
d) Absent peristalsis
e) Diarrhea

28. Which of the following is a major symptom of acute peritonitis?


a) Vomiting
b) Abdominal pain
c) Melaena
d) Stop offlatulence and defecation
e) Muscular defense

4
Test 5 Answers

Test 6

1. How many arteries supply the thyroid gland?

a) 2
b)3
c) 4 or 5
d)6

2. Which is the major structural element of the thyroid gland?


a) Colloid
b) Follicle
c) Triiodthyronine
d) Lobulus
e) Thyroxin

3. What is the main cause of endemic gaiter in some regions?


a) Family history
b) Iodine deficiency in soil, water and biosphere in some regions
c) High radioactivity levels in some regions
d) Chemical and heavy metal environmental pollution

4. Which are the major types of gaiter?


a) Nodular
b) Medullar
c) Mixed
d) Parenchymatous
e) Alveolar

5. How many types of esophageal diverticula are there depending on the localizatio n?
a) Cervical, thoracic and stomach diverticula
b) Pharyngoesophageal, tracheal biffurcation, epiphrenic diverticula
c) Epiphrenic, Zenker's, Meckel's diverticula

6. Which is the most common type of esophageal atresia?


a) Blind end of the proximal part of the esophagus and communication between the distal part b) of the esophagus
and the trach ea or the right bronchus
c) Proximal part of the esophagus communicates with the trachea, blind end of the distal part of the esophagus
d) Blind ends of both pro ximal and distal part of the esophagus, no communicatio n with the trachea
e) Proximal and distal part of the esophagus communicate both with the trachea

7. What is Water syndrome?


a) Multiple congenital malformations - esophageal atresia, vertebral, renal, cardiac,ano-rectal anomalies
b) Abscense of a diaphragm dome and lung aplasia, GIT disorders
c) Cystic lung disease and recurrent respirato ry infections

1
Test 5 Answers

8. Which is the first symptom in esophageal atresia?


a) Acute cardiovascular insufficiency
b) Cyanosis
c) Abdominal distension
d) Hypersalivation, recurrent foamy secretions from the nose and the mouth after aspiration

9. What would be your therapeutic approach to a 67-year-old patient with acute gangrenous cholecystitis and local
peritonitis?
a) Conservative therapy due to the age of the patient
b) Surgical treatment if conservative one is not effective
c) Depends on the stage of the disease
d) Delayed surgery
e) Emergency surgery

10. What is the treatment for a 77-year-old patient with destructive cholecystitis, acute peritonitis and
decompensated heart failure?
a) Laparoscopic cholecystectomy
b) Conventional cholecystectomy
c) Cholecystostomy
d) Percutaneous transhepatic cholangiography
e) Conservative treatment

11. Which of the following symptoms is not a sign of purulent cholangitis?


a) High body temperature
b) Septic fever
c) Bowel paresis
d) Pain in the right hypochondriac area
e) Obstructive jaundice

12. Which is the best and the safest diagnostic method for asymptomatic choledocholithiasis and papillary
stenosis?
a) Peroral cholecystography
b) Intravenous cholecystocholangiography
c) Laparoscopic cholecystocholangiography
d) Endoscopic retrograde cholangiopancreatography
e) Percutaneous transhepatic cholangiography

13. What does Ruggi-Parlavecchio femo ral hernia repair technique include?
a) Elimination of the inguinal canal
b) Suturing the inguinal to the pectineal ligament
c) lnguinalisation of the femoral hernia and elimination of both inguinal and femoral canal
d) No n of t he above
e) All of the above

14. Does surgica I treatment of femoral hernia include using a mesh?

2
Test 5 Answers

a) Yes
b) No

15. Could an abdominal approach be used in the surgical treatment of femoral hernia?
a) Yes
b) No

16. Could femoral hernia be treated laparoscopically?


a) Yes
b) No

17. Appendiceal artery is a direct branch of which of the following arteries?


a) Inferior mesenteric artery
b) Superior mesenteric artery
c) Descending branch of middle colic artery
d) lleocolic artery

18. What is the therapeutic approach to a 76-year-old patient who has acute myocardial infarction but there are
suspicions for acute destructive appendicitis?
a) Emergency surgery
b) Clinical observation and emergency surgery if there are signs of acute peritonitis
c) Administration of high doses of broad spectrum antibiotics and surgical treatment if there is no good effect of
the conservative therapy
d) All the answers are wrong

19. What does the right therapeutic behavior include if you are suspicious for acute appendicitis in a patient?
a) Urgent hospitalization
b) Administration of spasmolytic medications and another physical examination in 4-6h
c) Control total blood count test and measuring the body temperature in the next 12-24h
d) Anti inflammatory medications and control physical examination the next day
e) Observation in ambulatory care-sensitive conditions and hospitalization if the general condition worsens

20. Which of the following conditions requires abdominal cavity tamponade in acute appendicitis?
a) Diffuse capillary bleeding from the appendiceal area
b) 3d stage obesity
c) Gangrenous appendicitis
d) Periappendiceal abscess
e) Local peritonitis

21. Which of the following conditions are complications of Hirschprung's disease?


a) Intestinal obstruction
b) Rectorrhagia
c) Acute peritonitis
d) Intestinal fistulas

3
Test 5 Answers

22. What does treatment of Hirschprung's disease include?


a) Conservative therapy with cristaloid solutions and spasmolytic medications
b) Surgical treatment including resection of the aganglionic part and the dilated part of the sigmoid colon
c) Diet and enemas

23. Which of the following diagnostic methods are not used in Hirschprung's disease?
a) Colonoscopy
b) lrigography
c) Swenson's procedure
d) Manometry of the anal sphincter's tonus
e) Barium sulphate x-ray

24. What does treatment of chronic ulcerous colitis include?


a) Total parenteral nutrition
b) Total colectomy and ileostomy
c) Subtotal colectomy and ileostomy
d) All of the above
e) Non of the above

25. Which of the following conditions could not lead to acute peritonitis?
a) Meckel's diverticulum perforation
b) Crohn's disease
c) Papilla Vater stenosis
d) Richter's type incarceration
e) Acute intestinal obstruction

26. Which of the following symptoms is not typical for subdiaphragmatic abscess?
a) lnspiratory pain in the upper part of the abdomen and the right thoracic half
b) Pain in compressio n of the low er ribs
c) Septic fever
d) Kloiber glasses
e) Extended liver dullness

27. Which of the following symptoms is not typical for a subdiaphragmatic abscess?
a) Limited respiratory movements of the affected chest half
b) Elevated diaphragm
c) Reactive pleural effusion in the affected che st half
d) Pain irradiating to the supraclavicular space
e) Diarrhea

28. What is the treatment of subdiaphragmatic abscesses?


a) Conservative
b) Extraperitoneal drainage
c) Laparoto my and tampo nade of the abd ominal cavity
d) Abscess puncture under ultrasound control e) All of the above

4
Test 7 Answers

Test 7

1. Which are the atypical types of gaiter?


a) Retrosternal
b) Endothoracic
c) lntratracheal
d) Pulmonary
e) At the base of the tongue
f) Prescalene

2. What is the Grave's disease typical clinical triade?


a) Goiter
b) Hypothyroid ism
c) Exophtalmos
d) Tachypnea
e) Hyperpirexia
f) Tachycardia

3. Which are the typical signs in the clinical manifestation of Grave's disease?
a) Higher cardiac minute volume
b) Dry skin
c) Sweating
d) High irritability
e) Mental and physical apathy
f) Tremor of the upper extremities

4.Which are the main eye sympto ms in Grave's disease?


a) Rovsing symptom
b) Stellwag symptom
c) Kocher symptom
d) Bazedow symptom
e) Sitkovskiy symptom
f) Moebius symptom
g) Graefe symptom

5. How could esophageal atresia be diagnosed?


a) Barium sulfate X-ray
b) Water-soluble contrast material X-ray
c) CT-scan
d) MRI
e) Nasogastral tube

6. What is the treatm ent of esophageal atresia in a newborn?


a) Elective surgery
b) Emergency surgery

1
Test 7 Answers

c) Emergency surgery in a specialized hospital


d) Conservative treatment

7. What is the surgical treatment of esophageal atresia?


a) One-stage surgery
b) Two-stage surgery
c) All of the above

8. In which cases two-stage surgical procedures are indicated in esophageal atresia?


a) If there is a esophagotracheal fistula
b) If the distance between the proximal and the distal part of the esophagus is more than 2 cm
c) If there is a delay in the diagnosis

9. What kind of surgical procedure is indicated in a patient operated for cholelithiasis, common bile duct lithiasis
and purulent cholangitis?
a) Cholecystectomy and drainage of the subhepatic space
b) Cholecystostomy
c) Cholecystectomy and drainage of the common bile duct through the cystic duct stump
d) Cholecystectomy, choledochotomy and drainage of the common bile duct by a T-tube drain
e) All of the above

10. What is the right therapeutic approach to a patient with acute phlegmonous cholecystitis and 48h of non
effective conservative therapy?
a) Laparoscopy
b) Intensifying of the conservative therapy
c) Emergency surgery
d) Antibiotic therapy
e) Percutaneous transhepatic drainage of the common bile duct

11. Which of the following is not a type of common bile duct drainage?
a) T-tube drainage
b) Vishnevsky drainage
c) Spasokukotsky drainage
d) Halsted drainage
e) Non of the above

12. Which of the following is not a cholelithiasis complication?


a) Acute pancreatitis
b) Obstructive jaundice
c) Duodenal stasis
d) Small bowel obstruction
e) Purulent cholangitis

13. In which of the following groups femoral hernia happens more often?
a) In men

2
Test 7 Answers

b) In children
c) In men and children
d) Non of the above

14. What could happen after a femoral hernia incarceration?


a) Incarceration of the antimesenteric part of the bowel wall
b) Bowel wall necrosis
c) Peritonitis
d) All of the above
e) Non of the above

15. Is it possible not to have an intestinal obstruction as a clinical sign of incarcerated femoral hernia?
a) Yes
b) No

16. Could femoral hernias become big in size?


a) Yes
b) No

17. In which of the following cases appendectomy is contraindicated in acute appendicitis?


a) If there is appendiceal infiltrate
b) Acute myocardial infarction
c) Pregnancy in 30-40 gestational week
d) Novocain allergy
e) Coagulation status disorders

18. How could acute appendicitis be differentiated from kidney colic pain?
a) Administration of spasmolytics
b) Administration of opioid analgesics
c) Urine analysis
d) Chromocystography or excretory urography
e) Angiography of the renal vessels

19. Which of the following symptoms are typical for perforation of the appendix?
a) Free gas in the abdominal cavity
b) Reduction of the circulatory blood volume
c) Sudden intensification of the abdominal pain
d) Muscular defense of the abdominal wall
e) Positive Blumberg sign

20. What is the primary cause of gangrenous appendicitis?


a) Thrombosis of the inferior mesenteric artery
b) Stenosis of the ileocolic artery
c) Non specific arteriitis of the visceral branches of the aorta
d) Bacterial infection

3
Test 7 Answers

e) Thrombosis of the appendiceal artery

21. Which of the intestinal layers is affected in chronic ulcerous colitis?


a) Serosa
b) Muscular layer
c) Mucosa
d) All of the above

22. Which are the most common complications of chronic ulcerous colitis?
a) Hemorrhage and abscess formation
b) Perforation
c) Intestinal obstruction
d) All of the above

23. Conservative treatment of chronic ulcerous colitis includes:


a) Non steroid anti-inflammatory drugs
b) 5-ASA medications
c) Antibiotics
d) Pain-killers

24. Which of the following complications of chronic ulcerous colitis does not require surgical treatment?
a) Hemorrhage
b) Perforation
c) Malignancy
d) Toxic megacolon
e) Electrolyte imbalance

25. What kind of peritonitis do tuberculosis and abdominal typhus cause?


a) Primary
b) Secondary
c) Tertiary

26. What kind of peritonitis does hallow organs perforation cause?


a) Primary
b) Secondary
c) Tertiary

27. Diffuse peritonitis affects:


a) One abdominal area
b) Two abdominal areas
c) Three abdominal areas

28. When does the toxic stage of the acute peritonitis start?
a) Until the 24 th hour
b) 24th - 72 hours c) After 72 hours

4
Tests Answers

1.Which are the complications in Grave's disease?


a) Portal vein thrombosis
b) Heart attack
c) Atrial fibrillation
d) Compression of the major neck structures
e) Discoordination of the active movements

2. What is the conservative treatment for Grave's disease?


a) Treatment of tachycardia and rhythm disorders
b) Thyrostatic therapy with inorganic iodine
c) Thyroxine therapy

3. What is the difference between benign and malignant turners of the thyroid gland?
a) High mitotic activity in benign turners
b) Smooth surface of the malignant turners
c) Presence of a capsule in benign tumors
d) Mature thyroid cells as a sign of malignisation
e) Lower hormonal activity in thyroid adenomas

4. Which of the following are considered precancerous conditions for thyroid gland carcinoma?
a) Thyroid gland adenoma
b) Thyrotoxic adenoma
c) Goiter
d) Ridl thyroiditis
e) Thyrotoxicosis

5. What are the major diagnostic tests for thyroid gland carcinoma?
a) Tumor markers
b) Neck X-ray
c) Thyroid gland ultrasound and biopsy
d) CT-scan of head and neck
e) Scintigraphy

6. What is the origin of Zenker's diverticula?


a) Cervical part of the esophagus
b) Middle third of the esophagus
c) Distal third of the esophagus

7. Which are the most common symptoms in Zenker's diverticula?


a) Dysphagia and regurgitation
b) Heartburn and pain in the epigastrium
c) Heaviness and pain in the chest

8. Chest pain and stenocardia complaints are typical for:


a) Cervical diverticula

1
Tests Answers

b) Biffurcation diverticula
c) Epiphrenic diverticula
d) All of the above

9. What is the definition of esophageal achalasia?


a) Esophageal neoplastic disease with liver metastasis
b) Neuromuscular esophageal disease with disfunction of the lower esophageal sphincter
c) Neuromuscular disease of the pylorus and the stomach wall

10. Which of the following is a determining factor of the therapeutic approach in a patient with acute
phlegmonous cholecystitis?
a) Presence of acute peritonitis
b) Presence of concomitant diseases
c) Age of the patient
d) Surgeon's qualification
e) Presence of stones in the gallbladder

11. What is the therapeutic approach to a 53-year-old patient with obstructive jaundice due to a stone in the
common bile duct and acute liver insufficiency?
a) Emergency surgery and external drainage of the bile ducts
b) Emergency surgery and hemosorption in the postoperative period
c) Emergency surgery and external drainage of the thoracic duct
d) Emergency surgery after hemosorption
e) Emergency endoscopic papillosphincterotomy and hemosorption and surgical treatment after jaundice
reduction

12. What is the therapeutic approach to a 69-year-old patient with acute phlegmonous cholecystitis in a fair
general condition, heart rate 90bpm, soft abdominal walls upon palpation, pain in the right hypochondriac area
and negative Blumberg sign?
a) Emergency surgery
b) Conservative treatment including antibiotics
c) Conservative treatment and emergency surgery after 24h if there is no effect of the therapy
d) Cholecystectomy after preoperative preparation
e) It depends on the presence of stoned in the gallbladder and the bile ducts

13. Patients with biliary colic pain due to stones in the gallbladder are indicated for?
a) Emergency cholecystectomy
b) Conservative treatment
c) Antienzymatic therapy
d) Emergency surgery after colic pain reduction
e) Laparoscopic cholecystectomy

14. What is the definition of omphalocele?


a) Congenital umbilical hernia
b) Non obliterated omphaloenteric duct

2
Tests Answers

c) Non obliterated urachus

15. What is the treatment for congenital umbilical hernias?


a) Conservative
b) Elective surgery
c) Emergency surgery

16. What is the treatment for a congenital umbilical hernia with a small defect of the abdominal wall?
a) One stage surgery
b) Multiple stage surgery
c) Conservative treatment

17. What is the treatment for a congenital umbilical hernia with a big defect of t he abdominal wall?
a) Two stage surgery
b) One stage surgery
c) Conservative treatment

18. Which of the following symptoms are typical for phlegmonous appendicitis?
a) Blumberg sign
b) Bartomier-Michelson sign
c) Kocher-Volkovich sign
d) Rovsing sign
e) Murphy sign

19. How could acute appendicitis be differentiated from perforated duodenal ulcer?
a) Upper endoscopy
b) Abdominal X-ray
c) Abdominal ultrasound examination
d) Laparoscopy
e) Barium sulphate image study of the stomach

20. Acute appendicitis is an inflammatory processes affecting:


a) Caecum
b) Terminal ileum
c) lleocecal valve
d) Vermiform process

21. What does ileocecal bowel segment include?


a) Terminal ileum
b) Caecum and t he appendix
c) lleocecal valve
d) All of the above

22. Which are the indications for surgi cal treatment of chronic ulcerous co litis?
a) Toxic megacolon

3
Tests Answers

b) Perforation
c) Hemorrhage
d) Non effective conservative therapy
e) All of the above

23. Which of the following microorganisms causes pseudomembranous colitis?


a) Enterococcus fecalis
b) Bacteroides spp.
c) Clostridium perfringens
d) Clostridium difficile
e) E.coli

24. Which are the most complications of diverticulosis?


a) Diverticulitis
b) Hemorrhage
c) Perforation
d) Intestinal obstruction
e) Fistulas

25. Which of the following diagnostic methods could be used in colon polyps?
a) Colonoscopy
b) Virtual colonoscopy
c)MRI
d) All of the above

26. After leakage of which of the following contents the patient could develop aseptic peritonitis?
a) Bile juice
b) Stools
c) Pancreatic juice
d) Urine

27. Which of the following symptoms are typical for diffuse peritonitis?
a) Muscular defense
b) Positive Courvoisier sign
c) Positive renal sucussion
d) Positive Blumberg sign

28. Which of the following diagnostic methods is most informative in determining the T and N status in patients
with rectal carcinoma?
a) CT- scan
b) Rectoscopy
c) MRI
d) Digital rectal examination
e) Transanal ultrasound examination

4
Test9 Answers

1. Which of the following types are the most malignant tumors of the thyroid gland?
a) Small cell carcinoma
b) Follicular
c) Alveolar
d) Squamous
e) Gaint cell carcinoma
f) Papillary

2. Which are the possible complications after thyroid gland surgery?


a) Laryngeal nerve injury
b) Trigeminal nerve injury
c) Hemorrhage
d) Gas embolism
e) Heart attack

3. Which is the most common histological type of thyroid gland carcinoma?


a) Follicular
b) Papillary
c) Medullar
d) Non differentiated

4. What is the most appropriate therapeutical approach to a patient with a 2 cm "cold" nodule in the left thyroid
lobe and histological result of malignant cells after fine needle aspiration biopsy?
a) Radioiodine ablation
b) Thyroidectomy
c) Lobectomy
d) Enucleoresection

5. What is the first method of choice for treatment of a 76-year-old patient with histological verification of
esophageal carcinoma, 27cm away from the oral cavity, liver metastasis, dysphagia and total obstruction of the
esophageal lumen?
a) Esophageal resection by two surgical approaches
b) Feeding gastrostomy
c) Self-dilating tube insertion
d) Adjuvant chemotherapy

6. Assessment of respectability of esophageal carcinoma depends on:


a) The histological verification
b) The histological verification and staging
c) The type of the tumor and the age of the patient

7. What is Trousseau syndrome?


a) Purple spots around the umbilicus
b) Migrating thrombophlebitis
c) Thrombophlebitis of the lateral thoracic vein
d) Enlarged no n painful gallbladder

1
Test9 Answers

8. Which is the upper border of the anterior abdominal wall?


a) Xiphoid process
b) Left costal margin
c) Right costal margin
d) All of the above
e) None of the above

9. Which of the following diagnostic methods is not a method of cgoice in obstructive jaundice?
a) Liver enzymes test
b) Infusion cholangiography
c) Laparoscopy
d) Endoscopic retrograde cholangiopancreatography
e) Percutaneous transhepatic cholangiography

10. Which is the first symptom of acute cholecystitis?


a)Fever
b)Vomiting
c) Pain in the right hypochondriac area
d) Diarrhea
e) Discomfort in the epigastrium

11. Biliary colic pain beginning is:


a) Sudden and sharp
b) Gradual
c) After long period of starving
d) After hypothermia

12. Which of the following drug is contraindicated in acute and chronic cholecystitis?
a) Papaverin
b) Morphine hydrochloride
c) Drotaverine
d) Atropine sulphate
e) Spasmalgon

13. What is the treatment for an umbilical hernia in children?


a) Conservative
b) Surgical
c) Surgical after unsuccessful conservative treatment after the 1st year of the child

14. Which are the clinical features of an umbilical hernia in adults?


a) Small in size, no signs of incarceration, spontaneous recovery
b) Big in size, frequent incarcerations, surgical treatment
c) Big in size, rare incarcerations, surgical treatment

lS. What is the name of the surgical procedure in which the hernia ring of an umbilical hernia is sutured ?

2
Test9 Answers

a) Mayo technique
b) Sapozhko-Piccoli technique
c) Mesh hernia repair

16. What is the non obliterated omphaloenteric duct?


a) Fistula between the small intestine and the umbilicus
b) Meckel diverticulum
c) Fistula between the urine bladder and the umbilicus

17. Appendix is a rudimentary organ and is formed by:


a) Terminal ileum wall
b) lleocecal valve
c) Caecum wall
d) Transverse colon wall

18. What is the function of the appendix?


a) Secretory
b) Contractile
c) Lymphocytic
d) All of the above
e) It has no functions

19. Which are infection's spreading ways in acute appendicitis?


a) Haematogenous and contact
b) Lymphogenous
c) Enterogenous
d) All of the above

20. Which of the following organs is also known as abdominal tonsil?


a) Terminal ileum
b) Caecum
c) lleocecal valve
d) Appendix

21. Which is the golden standard in the treatment of colon polyps ?


a) Conservative
b) Endoscopic polypectomy
c) Surgical polypectomy
d) Segmental resection of the affected part of the colon

22. What is the metastatic way o f colorectal cancer?


a) Lymphogenous
b) Hematogenous
c) Local invasion
d) Implantation
e) All of the above

3
Test9 Answers

23. Which of the following conditions is the most common precancerosis for colorectal cancer?
a) Skin nevus
b) Lynch syndrome
c) Colon polyposis
d) Peutz-Jeghers syndrome

24. Which is the most common symptom in left-sided colorectal cancer?


a) Constipation/ intestinal obstruction
b) Anemia
c) Headache
d) Pain

25. Which of the following is a contraindication for rectal resection in patients with cTl status?
a) cN+
b) High grade ea rcinoma
c) Infiltration of the deepest submucosal layer
d) All of the above

26. Which is the most minimally invasive surgical procedure that could be used in a 82-year-old patient with colic
abdominal pain, nausea and vomiting, stop offlatulence and defecation for three days, abdominal distension with
hypersonic sound upon percussion, decreased peristalsis, empty rectal ampulla upon digital rectal examination.
Concomitant diseases - myocardial infarction and coronary stent insertion, chronic obstructive pulmonary
diasease. Anesthesiologist consultation -ASA 4 . CT - scan data for tumor of the proximal rectum, dilated small and
large bowel loops, hydroaeric levels.
a) Emergency surgery, Hartmann procedure
b) Laparoscopy, double barrel proximal colostomy
c) Endoscopic stent insertion

27. Which of the following statements is false?


a) Short- and long-term oncology results in laparoscopic rectal resection in rectal carcinoma are equal to those
after conventional open surgery
b) Conversion to open surgery after laparoscopic recta l resection is more often that conversio n to open surgery
after laparoscopic colon cancer resection
c) Resectable carcinoma of the distal third of the rectum is a contraindication for laparoscopic resection
d) Previous abdominal surgery is not a contraindication for laparoscopi rectal resection

28 Transanal minimally invasive surgery is a method of choice in:


a) Rectal polyps
b) Rectal carcinoma
c) All of the above

4
Test 10 Answers

Test 10

1. What are the neuroprotective measures that should be taken towards laryngeal recurrent nerves during thyroid
surgery?
a) lntraoperative CT-navigation
b) lntraoperative neuromonitoring
c) Express histological examination
d) Using magnifying glasses

2. What is the most dangerous complication of thyroid gland surgery?


a) Acute intraoperative haemorrhage
b) Postoperative hypoparathyroidism
c) Dis- or aphonia
d) All of the above

3. What is the obligatory condition for considering a thyroid gland surgical procedure as minimally invasive one?
a) Using an endoscope
b) Incision to 3 cm
c) Preserving prethyroid muscles
d) All of the above

4. What does "near total thyroidectomy" mean?


a) Preserving 3-4g of the thyroid gland
b) Preserving small amount of thyroid tissue
c) lntracapsular blood vessel ligation
d) None of the above

5. Which areas does epigastrium include?


a) Left and right hypochondriac area, epigastric area
b) Right hypochondriac area, epigastric area, left lumbar area
c) Right lumbar area, epigastric area, left lumbar area
d) Right and left hypochondriac area, umbilical area
e) None of the above

6. Which areas does mesogastrium include?


a) Right and left hypochondriac area, epigastric area
b) Right hypochondriac area, epigastric area, left lumbar area
c) Right and left lumbar area, umbilical area
d) Right and left hypochondriac area, umbilical area
e) No ne of the above

7. Which areas does hypogastrium include?


a) Right and left hypochondriac area, suprapubic area
b) Right and left iliac area, suprapubic area
c) Right and left lumbar area, supra pubic area

1
Test 10 Answers

d) Right and left hypochondriac area, umbilical area


e) None of the above

8. Anterior abdominal wall consists of which of the following muscles?


a) External oblique muscle, transverse abdominal muscle, rectus abdominis muscle
b) Internal oblique muscle, transverse abdominal muscle, rectus abdominis muscle
c) External oblique muscle, internal oblique muscle, transverse abdominal muscle
d) External oblique muscle, Internal oblique muscle, rectus abdominis muscle
e) None of the above

9. Which of the following procedures is not used in extra hepatic bile ducts examination?
a) Palpation of the common bile duct
b) Choledochoscopy
c) lntraoperative cholangiography
d) Catheter probe examination of the common bile duct
e) Intravenous cholangiography

10. What is the treatment for a patient with obstructive jaundice due to choledoholithiasis?
a) Emergency surgery
b) Conservative treatment
c) Delayed surgery after preoperative preparation
d) Celiac trunk catheterization
e) Plasmapheresis

11. Which of the following is indicated in acute calculous cholecystitis?


a) Emergency surgery
b) Delayed surgery
c) Conservative therapy followed by elective surgery
d) Conservative treatment
e) All of the above

12. Courvoisier symptom is not typical for?


a) Acute calculous cholecystitis
b) Pancreatic head carcinoma
c) lndurative pancreatitis
d) Papillary tumor
e) Common bile duct tumor

13. What is the clinical manifestation of an non obliterated urachus?


a) Fistula between the urine bladder and the umbilicus
b) Fistula between the small intestine and the umbilicus
c) Fistula between the colon and the umbilicus

14. What is the definition of a Meckel diverticulum?


a) Partially non obliterated omphaloenteric duct

2
Test 10 Answers

b) Partially non obliterated urachus

15. Where is the localization of a Meckel diverticulum?


a) Ileum
b) Jejunum
c) Ascending colon
d) Transverse colon
e) Sigmoid colon

16. How many types of diaphragmatic hernias are there?


a) Traumatic
b) Non traumatic
c) Traumatic and non traumatic

17. How many types of acute appendicitis are there?


a) Superficial (catarrhal)
b) Destructive
c) Complicated
d) All of the above

18. What does destructive appendicitis mean?


a) Catarrhal
b) Appendiceal colic
c) Phlegmonous
d) Phlegmonous, gangrenous and perforated appendicitis

19. Which are the complications of acute appendicitis?


a) Periappendiceal infiltrate and abscess formation
b) Pylephlebitis and sepsis
c) lntrabdominal pus collections and peritonitis
d) All of the above

20. Which of the following factors determine the clinical manifestation of acute appendicitis?
a) Age, sex, general condition
b) Pathological changes in the appendix and other abdominal organs
c) Macroorganism resistance to infections and presence of any complications of the disease
d) All of the above

21. Which is the most common symptom in righ-sided colorectal cancer?


a) Constipation/intestinal obstruction
b) Anemia
c) Heada che
d) Pain

22. Which of the following procedures is a golden standard in the surgical treatment of rectal cancer?

3
Test 10 Answers

a) Rectotomy
b) Partia I rectal resection
c) Total mesorectal excision
d) Rectal resection

23. What are the first therapeutic measures in metastatic colorectal cancer?
a) Adjuvant chemotherapy
b) Surgical treatment
c) Neoadjuvant chemotherapy
d) Radiotherapy

24. Which is the surgical method of choice in treatment of cecal carcinoma?


a) Left hemicolectomy
b) Right hemicolectomy
c) lntermedium hemicolectomy
d) Subtotal colectomy

25. Which IBD does not affect the rectum?


a) Crohn's disease
b) Chronic ulcerative colitis

26. Where is the typical localization of hemorrhoids in a gynecological position?


a) 3 o'clock
b) 5 o'clock
c) 7 o'clock
d) 9 o'clock
e) 11 o'clock

27. Which anatomical feature is the border between internal and external hemorrhoids?
a) Dentate line
b) Anal orifice
c) Recto-sigmoid junction
d) Proximal rectal third
e) Anal levator muscle

28. Which of the following is a typical symptom in hemorrhoids?


a) Pain
b) Anal bleeding after defecation
c) Anal bleeding before defecation
d) Diarrhea
e) Anal itching

4
Test 11 Answers

Test 11

1. What is the criteria according to which we divide chest traumas to open and closed?l
a) Whether there is a chest bones injury
b) Whether there is a skin injury
c) Whether there is a pulmonary or pleural injury

2. What is the definition of penetrating chest trauma?


a) Skin injury
b) Thoracic fascia injury
c) Pleural injury
d) Pulmonary injury

3. What is the medical term for the presence of blood and air in the pleural cavity and shifting of the mediastinum
to the non affected side of the chest?
a) Tension chylopneumothorax
b) Haemothorax
c) Tension haemopneumothorax

4. What is the medical term for the presence of blood in the pleural cavity?
a) Pyothorax
b) Chylothorax
c) Haemothorax
d) Haemascos

5. How many walls does the inguinal canal have?


a) 1
b) 2
c) 3
d)S
e) None of the above

6. Which is the inferior border of the inguinal canal?


a) Inguinal ligament
b) Pectineal ligament
c) Transverse fascia
d) Internal oblique muscle
e) None of the above

7. Which of the following elements forms the anterior wall of the inguinal canal?
a) Internal oblique muscle aponeurosis
b) Transverse abdominal muscle aponeurosis
c) A and B
d) Rectus abdominis muscle aponeurosis
e) Non of the above

1
Test 11 Answers

8. Which of the following elements forms the superior wall of the inguinal canal?
a) Lower parts of the internal oblique muscle
b) Lower parts of the transverse abdominal muscle
c) A and B
d) Lower parts of the rectus abdominis muscle
e) All of the above

9. Which of the following is not typical in obstructive jaundice due to choledocholithiasis?


a) Hyperthermia
b) Increased levels of direct bilirubin
c) Increased levels of alkaline phosphatase
d) Increased levels of liver transaminases
e) Absence of stercobilin in the stools

10. Which of the following procedures is not used for diagnosing obstructive jaundice?
a) CT-scan
b) Intravenous cholecystocholangiography
c) Percutaneous transhepatic cholangiography
d) Endoscopic retrograde cholangiopancreatography
e) Abdominal ultrasound examination

11. Which of the following is not typical for acute obstructive cholangitis?
a) Jaundice
b)Fever
c) Small size of the liver
d) Leucocytosis
e) Liver enlargement

12. Which of the following is not typical for acute cholangitis?


a)Fever
b) Pain in the right hypochondriac area
c) Jaundice
d) Leucocytosis
e) Diarrhea

13. What is the localization of non traumatic diaphragmatic hernias?


a) Esophageal hiatus
b) Larrey and Morgagni space, Bochdalek' s fora men
c) All of the above

14. Where does hiatal hernia pass through?


a) Esophageal hiatus of the diaphragm
b) Larrey and Morgagni space
c) Bochdalek's foramen

2
Test 11 Answers

15. Where does parasternal diaphtragmatic hernia pass through?


a) Larrey and Morgagni space
b) Bochdalek's foramen
c)Esophageal h~tus

16. Where do posterolateral diaphragmatic hernias pass through?


a) Larrey and Morgagni space
b) Bochdalek's foramen
c)Esophageal h~tus

17. Which of the following is the major symptom of acute appendicitis?


a) Nausea and vomiting
b) Fever and increased heart rate
c) Diarrhea
d) Abdominal pain

18. What is the first localization of the pain in acute appendicitis?


a) In the epigastrium
b) Diffuse abdominal pain
c) In the ileocecal area
d) Around the umbilicus
e) Above the pubic symphysis

19. Which of the following signs are typical for acute appendicitis?
a) Kocher-Volkovich sign
b) Courvoisier sign
c) Rovsing sign
d) Murphy sign
e) Obraztsov sign (Psoas sign)

20. What is important for diagnosing acute appendicitis?


a) Localization of the pain in the beginning of the disease
b) Final localization of the pain
c) First and final localization of the pain
d) Pain localization is not related to the diagnosis
21. Which is the surgical method of choice in treatment of descending colon cancer?
a) Left hemicolectomy
b) Right hemicolectomy
c) lntermedium hemicolectomy
d) Subtotal colectomy

22. What is a combined trauma?


a) Traumatic injury of many organs in one anatomical cavity
b) Traumatic injury caused by a few traumatic agents at the same time

3
Test 11 Answers

c) Traumatic injury of organs in different anatomical cavities (thoracic and abdominal, central nervous system
and thoracic cavity)
d) Trauma to an organ with previous pathological changes

23. Which of the following conditions is a predisposing factor for a rupture of a hollow abdominal organ in closed
trauma?
a) If the organ is empty
b) If the organ is full
c) If the organ is mobile
d) If the organ is in a fixed position
e) If the organ is a tonic

24. Which of the following diagnostic methods is most informative in liver rupture?
a)ERCP
b) Upper endoscopy
c) Native abdominal X-ray
d) Contrast abdominal X-ray
e) Laparoscopy

25. Which of the following are typical complications of hemorrhoids?


a) Thrombosis and inflammation
b) Massive hemorrhage
c) Abscess and fissure formation, intoxication
d) Rectal carcinoma
e) Anal and rectal prolapse

26. What does surgical treatment of hemorrhoids include?


a) Ultrasound ligation
b) Morgan's procedure
c) Alcoholization
d) Whitehead's procedure
e) Liquid nitrogen therapy

27. What is the typical localization of anal fissure in gynecological position?


a) 3 o'clock
b) 6 o'clock
c) 7 o'clock
d) 9 o'clock
e) 11 o'clock

28. Which are the symptoms of anal fissure?


a) Pain during and after defecation
b) Mucus and pus anal discharge
c) Bleeding
d) Anal sphincter spasms e) Diarrhea

4
Test 12 Answers

Test 12

1. What is the definition of flail chest?


a) Three or more consecutive ribs or costal cartilages are fractured in two or more places and this segment has
an independent mobility
b) Three or more consecutive ribs or costal cartilages are fractured in two or more places and this segment doesn't
have an independent mobility
c) Shoulder complex paradoxal mobility

2. Which type of pneumothorax requires prehospital emergency care?


a) Open subtotal pneumothorax
b) Partial pneumothorax
c) Tension pneumothorax
d) Total pneumothorax without shifting of the mediastinum

3. What is the most common fracture in chest trauma?


a) Rib fracture
b) Clavicular fracture
c) Sternal fracture
d) Scapular fracture

4. What are the clinical and X-ray signs of tension pneumothorax?


a) Shadow on the affected chest half
b) Total collapse of the affected lung and shifting of the mediastinum to the contra lateral sideof the chest
c) Dull percussion sound of the affected side
d) No breathing sounds of the affected side

5. Which of the following elements forms the posterior wall of the ingunal canal?
a) Transverse fascia
b) Transverse abdominal muscle
c) Rectus abdominis muscle
d) Internal oblique muscle
e) External oblique muscle

6. How many rings does inguinal canal have?


a) Internal (deep)
b) Medium
c) External (superficial)
d)AandC
e) All of the above

7. Which artery is the medial border of the deep inguinal ring?


a) Inferior epigastric artery
b) Superficial epigastric artery
c) Superior epigastric artery

1
Test 12 Answers

d) Internal iliac artery


e) Internal pudenda I artery

8. Is there a femoral canal if there is no hernial sac in it?


a) Yes
b) No

9. What is the cause of intermittent obstructive jaundice?


a) Incarcerated stone in the terminal part of the common bile duct
b) Common bile duct tumor
c) Stone in the cystic duct
d) Stone in the common bile duct causing ventil type of obstruction
e) Stricture of the common bile duct

10. In which of the following cases an emergency surgery for choledocholithiasis is indicated?
a) In cystic duct occlusion
b) In biliopancreatitis
c) In perforative cholecystitis
d) In mechanical jaundice

11. Which of the following conditions is a choledocholithiasis complication?


a) Hydrops of the gallbladder
b) Empyema of the gallbladder
c) Obstructive jaundice and cholangitis
d) Chronic exacerbated hepatitis
e) Perforative cholecystitis and peritonitis

12. Which of the following is a method of choice in the therapeutic approach to a patient with chronic calculous
cho lecystiti s?
a) Litholytic medications intake
b) Microcholecystostomy
c) Lithotripsy
d) Cholecystectomy
e) Complex conservative treatment

13. What is the most important and constant symptom in congenital diaphragmatic hernias?
a) Cyanosis
b) Vo mits
c) Abd ominal distension

14. What organs disorders are typical for diaphragmatic hernias?


a) Abdo minal o rgans
b) Thoracic organs
c) Abdominal and thoracic o rgans

2
Test 12 Answers

15. What are the physical examination findings in incarcerated diaphragmatic hernia?
a) Tympanic sound in the affected half of the chest
b) Decreased or absent breathe sounds and presence of peristalsys in the affected half of the chest
c) All of the above

16. What is the treatment for incarcerated diaphragmatic hernia?


a) Elective surgery
b) Emergency surgery
c) Conservative treatment

17. Which are the cardinal symptoms of acute appendicitis?


a) Pain upon palpation
b) Muscular defense
c) Positive Blumberg sign
d) All of the above

18. Which of the following symptoms is always present in acute appendicitis?


a) Muscular defense
b) Positive Blumberg sign
c) Spontaneous and palpation-induced pain
d) Increased heart rate

19. High leucocyte rate in acute appendicitis is:


a) Determinant factor the diagnosis
b) Important factor for the diagnosis
c) It is not related to the diagnosis
d) Additional factor for the diagnosis

20. Muscular defense and positive Blumberg sign in acute appendicitis are:
a) Constant symptoms in all the types of acute appendicitis
b) Always observed in the catarrhal forms
c) Always observed in the destructive forms
d) Rarely observed in catarrhal and observed in 70-92% of the destructive forms

21. Which of the following diagnostic methods is most informative in urine bladder rupture?
a) Laparoscopy
b) Abdominal ultrasound
c) Transrectal ultrasound
d) Retrograde contrast cystography
e) Urine analysis

22. In which of the following cases abdominal ultrasound examination is an informative diagnostic method?
a) Stomach rupture
b) Small bowel rupture
c) Urine bladder rupture

3
Test 12 Answers

d) Rupture of parenchymatous abdominal organs

23. What is the clinical manifestation of hemobilia?


a) Positive Blumberg sign
b) Pain in the right hypochondriac area
c) GIT bleeding
d) Intermittent hyperbilirubinemia
e) Positive Cullen's sign

24. Which of the following diagnostic methods could be used in hemobilia?


a) Laparocentesis
b) Laparoscopy
c) er-scan
d) Abdominal ultrasound
e) Angiography

25. Which are the branches of inferior mesenteric artery?


a) Left colic artery
b) Right colic artery
c) Middle colic artery
d) Middle rectal artery

26. How could internal hemorrhoids be classified if they prolapse out the anus without spontaneous reduction?
a) First stage
b) Second stage
c) Third stage

27. What is the surgical treatment of third and fourth stage hemorrhoids?
a) Ultrasound ligation (THD; HAL-RAR)
b) lnfrared photocoagulation
c) Longo's procedure - hemorrhoidectomy by a circular stapler
d) All of the above

28. What is the therapeutic method of choice in acute anal fissure?


a) Excision and primary suture
b) Lateral sphincterotomy
c) Botulinum toxin therapy
d) Laxatives and dressings

4
Test 13 Answers

1. Which are the typical features of pulmonary contusion?


a) It is visible on X-ray picture right after the trauma
b) Treatment of this condition is predominantly surgical
c) It leads to a worse arterial blood gas test
d) It developes in severe chest traumas
e) It is visible on Cl-scan of the chest

2. What are the typical features of flail chest?


a) Impaired respiratory mechanics due to segmental paradoxal movement of the chest
b) Presence of pulmonary contusion with impaired gas exchange
c) Clinical and laboratory signs of respiratory insufficiencyM
d) Treatment includes immobilization of the fractured ribs
e) Treatment includes positive pressure mechanical ventilation

3. What complication could develop in a 16-year-old patient with a puncture chest trauma, poor general condition,
severe dyspnea, tachycardia, opening of the wound during inspirium and collapsing during expirium?
a) Tension pneumothorax
b) Pleural haemorrhage
c) Major bronchus injury

4. What is the urgent therapeutical measure in tension pneumothorax?


a) Transforming it into open one
b) Transforming it into closed one
c) Ointment wound bandage

5. Which of the following elements forms the anterior wall of the femoral canal?
a) Pectineal ligament
b) Inguinal ligament
c) Arcuate ligament
d) And B
e) Non of the above

6. Which of the following elements forms the lateral wall of the femoral canal?
a) Inguinal ligament
b) Femoral artery
c) Femoral vein
d) Femoral nerve
e) All of the above

7. Which of the following elements forms the medial wall of the femoral canal?
a) Pectineal ligament
b) Inguinal ligament
c) Arcuate ligament
d)Aand B
e) All of the above

1
Test 13 Answers

8. Which of the following elements forms the posterior wall of the femoral canal?
a) Pectineal line of the pubis
b) Inguinal ligament
c) Arcuate ligament
d)BandC
e) Non of the above

9. What is the diagnosis of a 57-year-old patient with moderate pain in the right hypochondriac area which
irradiates to the right scapula, history data for chronic calculous cholecystitis, no laboratory changes, no jaundice,
painful palpable gallbladder, no fever?
a) Gallbladder empyema
b) Pancreatic head carcinoma
c) Hydrops of the gallbladder
d) Acute perforative cholecystitis
e) Liver echinococcosis

10. Which of the following is typical for acute cholangitis?


a) Pain
b) Jaundice
c) Fever
d) Melaena

11. Cholangitis is a complication of which disease?


a) Duodenal ulcer
b) Choledocholithiasis
c) Bile duct stricture
d) Calculous cholecystitis

12. What is the aim of purulent cholangitis treatment?


a) Normalizing the intraluminal pressure of the common bile duct
b) Anti infective therapy
c) Desobstruction of the bile ducts
d) Biliodigestive by pass

13. What kind of surgical approach is used for surgical treatment of diaphragmatic hernias?
a) Abdominal
b) Thoracic
c) Thoracoabdominal

14. How many types of hiatal hernia are there?


a) Sliding
b) Paraesophageal
c) Sliding, paraesophageal and mixed

2
Test 13 Answers

15. Which is the most common type of congenital diaphragmatic hernia?


a) Relaxation of the diaphragm
b) Dome hernia
c) Larrey and Morgagni space hernia
d) Bochdalek foramen hernia
e) Phrenopericardial hernia

16. Which is the major diagnostic method for diaphragmatic hernias?


a) X-ray
b) CT scan
c) Abdominal ultrasound
d) MRI

17. Which of the following conditions is included in the differential diagnosis of acute appendicitis?
a) Acute gynecological conditions
b) Diseases of extra peritoneal organs
c) Right-sided kidney colic
d) All the disease leading to acute abdomen and some extra peritoneal organ diseases

18. Retrocecal appendicitis could imitate:


a) Acute gynecological condition
b) Acute cholecystitis
c) lntussusception
d) Right-sided kidney colic

19. What is the treatment for acute appendicitis?


a) Local ice pack placement and antibiotic therapy
b) Physiotherapy
c) Emergency surgery
d) Surgical treatment if the conservative therapy is not effective

20. Subhepatic appendicitis could imitate:


a) Acute cholecystitis
b) Acute pancreatitis
c) Kidney colic

21. In which of the following cases laparoscopy is contraindicated?


a) Pancreatic trauma
b) Rupture of hollow abdominal organs
c) Retroperitoneal hematoma
d) Diaphragmatic rupture
e) lntraabdominal bleeding

3
Test 13 Answers

22. What is the cause of bleeding in a 28-year-old patient operated for liver rupture 5 days ago, who is now with
pain in the right hypochondriac area and irradiation to the right scapula, vomiting, melaena, Hb level drop from
l0Sg/1 to 69g/l, HR 104bpm?
a) Erosive gastritis
b) Stress ulcerations
c) Duodenal ulcer
d) Portal hypertension
e) Hemobilia

23. Which of the following surgical procedures are not methods of choice for hemostasis in liver hemorrhage?
a) Resection
b) Transhepatc drainage
c) Suture of the liver parenchyma
d) Ligation or embolisation of arterial blood vessel
e) Packing

24. Which of the following features are intraoperative findings in retroduodenal rupture?
a) Retroperitoneal hematoma
b) Retroperitoneal infiltration with bile juice
c) Gas in the soft tissues
d) Retroperitoneal food materials
e) Pancreatic juice

25. lschiorectal spaces connect above the anococcigeal ligament and form the deep retroanal space.
a) True
b) False

26. Which of the following conditions could be observed in anal and rectal prolapse?
a) Prolapse of the serosa layers of the anus and rectum
b) Prolapse of rectal mucosa
c) Massive hemorrhage
d) Incontinence
e) Sharp pain during defecation

27. What is the treatment of anal and rectal prolapse in children?


a) Tissue glue injection
b) Resection of the prolapsed segment
c) Alcoholization
d) Duval' s procedure
e) Manual reduction of the prolapsed segment and suturing

28. What is the surgical method of choice in treatment of anal and recta l prolapse in adults?
a) Tissue glue injection
b) Resection of the prolapsed se gment
c) Alcoholization d) Duval's procedure e) Manual reduction of the prolapsed segment and suturing

4
Test 14 Answers

1. What is the predisposing risk factor for developing acute puerperal mastitis?
a) Milk stasis and nipple trauma
b) Hereditary factors
c) Presence of milk tooths of the baby
d) Staphylococcal presence in the mother

2. How many types of mammary gland abscesses are there?


a) Superficial ( premammary, subareolar)
b) lntramammary
c) Retromammary
d) All of the above

3. How could intra mammary abscesses be divided?


a) Interstitial
b) Parenchymatous
c) Retromuscular
d) Subareolar

4. What is the treatment of acute puerperal mastitis?


a) Conservative
b) Puncture
c) Incision and drainage
d) Mastectomy

5. Is there any subcutaneous tissue in the umbilical area?


a) Yes
b) No

6. Which are the risk factors for hernial appearance?


a) Cachexia
b) Ascites
c) Many abdominal surgical procedures
d) All of the above
e) Non of the above

7. Which are the risk factors for hernial appearance?


a) Cachexia
b) Ascites
c) Many abdominal surgical procedures
d) All of the above
e) Non of the above

8. Which hernia types are external?


a) Inguinal, femoral ,Spigelian
b) Inguinal, femoral, diaphragmatic

1
Test 14 Answers

c) Diaphragmatic and femoral


d) Inguinal and diaphragmatic
e) Non of the above

9. Which of the following diagnostic methods are most informative for acute cholangitis?
a) Abdominal X-ray
b)ERCP
c) Abdominal ultrasound
d) Gastroduodenal endoscopy
e) Urine analysis

10. How many types of liver traumas are there according to the liver trauma classification?
a) Superficial
b) Transcapsular
c) Subcapsula
d) Lobar
e) Central

11. How many stages of liver trauma are there depending on the severity of the injury?
a) 2 stages
b) 3 stages
c) 4 stages
d) 5 stages

12. Which are the clinical features of liver trauma?


a) Shock
b) Abdominal pain
c) Muscular defense
d) Diarrhea

13. Which are the complications of ulcer disease?


a) Perforation
b) Penetration
c) Malignancy
d) Haemorrhage
e) Stenosis
f) Malabsorbtion and maldigestion

14. Which of the following statements a bout pyloric stenosis is true?


a) Chronic complication of ulcer disease
b) It could be due to malignancy
c) It leads to dyselectrolytemia
d) Patients with this condition gain weight

15. Which of the following statements a bout the congenita I pyloric stenosis is true?

2
Test 14 Answers

a) The disease affects male newborns and manifests after 20th day of birth
b) Clinical manifestation includes constant forceful vomits after feeding
c) It is a complication of ulcer disease
d) There is a positive Blumberg sign

16. Which of the following statements a bout the treatment of congenita I pyloric stenosis is true?
a) Conservative treatment is leading
b) Surgical treatment includes Fredet -Weber- Ramstedt procedure
c) Pyloromyotomy aims opening of all the three layers of the esophagus

17. In which of the following appendiceal positions acute appendicitis could imitate acute enterocolitis?
a) Pelvic
b) Re trope ritonea I
c) Subhepatic
d) Mesocecal

18. Which of the following diagnostic methods are not useful for diagnosing acute appendicitis?
a) Abdominal palpation
b) Blood samples
c) Abdominal ultrasound examination
d) Pneumocolonoscopy

19. Which of the following symptoms are typical for destructive appendicitis?
a) Hydroaeric levels on abdominal X-ray
b) Free gas under the diaphragmatic domes
c) Muscular defense and positive Blumberg sign
d) More than 1 degree difference between rectal and axillary temperature

20. Which are the complications of a destructive appendicitis?


a) Periappendiceal infiltrate or abscess
b) lntraabdominal abscess, Douglas-pouch abscess, subphrenic abscess
c) Acute purulent peritonitis, pylephlebitis, sepsis
d) All of the above

21. Traumatic injury of which of the following organs does not lead to hemoperitoneum?
a) Spleen
b) Mesenterium
c) Pancreas
d)Liver

22. What does treatment of superficial capsular lesion of the spleen include?
a) Surgical treatment, splenectomy
b) Clinical observation
c) Surgical treatment if there is acute hemorrhage
d) Puncture and decompression of the hematoma

3
Test 14 Answers

23. What is the most appropriate therapeutic behavior if there is a cyanotic bowel loop with decreased peristalsis
as a intraoperatve finding during a surgical procedure for adhesive intestinal o bstruction?
a) Intestinal resection
b) Ad ministration of antiholinesterases to help bowel motility
c) Novocain solution application in the root of mesentery
d) Warming the cyanotic bowel loop*
e) Nasogastric tube insertion

24. What is the optimal surgical procedure in a 70-year-old patient with sigmoid colon volvulus for more than 24h,
necrosis and colon dilation as an intraoperative finding?
a) Double barrel sigmoid colostomy
b) Sigmoidectomy and primary end-to-end anastomosis
c) Sigmoidectomy and primary side-to-side anastomosis
d) Sigmoidectomy and colostomy
e) Non of the above

25. What does acute paraproctitis mean?


a) Ana I fissure
b) Perianal abscess
c) Hemorrhoids
d) Anal fistula
e) Fournier's gangrene

26. What is the major cause of acute paraproctitis?


a) Anal glands infection
b) Anal sphincter infection
c) Perianal soft tissue infection
d) Anal mucosa infection
e) Hair follicles infection

27. What is the term for acute paraproctitis which spreads above the anal levator muscle?
a) lschiorectal abscess
b) Horseshoe-like abscess
c) Pelviorectal abscess
d) Anal fistula
e) Submucosal abscess

28. What is the term for an acute paraproctitis which spreads t o the scrotal space?
a) Anal fistula
b) Fournier's gangrene
c) Horseshoe-like abscess
d) Epididimitis and o rchitis
e) Anal fissure

4
Test 15 Answers

1. What is the definitive treatment of chronic subareolar abscess of the mammary gland?
a) Subcutaneous mastectomy
b) Incision, lavage, drainage
c) Excision of the milk sinus and the surrounding affected tissues
d) Puncture

2. What is the fibrocystic breast disease?


a) Obligatory precancerosis
b) Requires radical surgical treatment
c) Asymptomatic disease, requires no treatment
d) Requires recurrent examinations and conservative treatment

3. Which of the following statements about the proliferative benign disease of the mammary gland is true?
a) This is an accidental finding after mammary gland surgery
b) There is a higher risk for developing breast cancer
c) Requires hormonal therapy
d) They are absolute indication for mastectomy

4. True-cut biopsy is:


a) Method for histological verification
b) Method for getting 4-8 tissue samples
c) Using a specific device
d) Must be performed under X-ray control
e) All of the above

5. What are the diaphragmatic hernias?


a) External hernias
b) Internal hernias
c) All of the above

6. Which is the most common type of hernia in men?


a) Femoral
b) Umbilical
c) Diphragmatic
d) Spigelian
e) Non of the above

7. Which is the most common type of hernia in women?


a) Femoral
b) Umbilical
c) Inguinal
d) Spigelian
e) Non of the above

8. In which sex femoral hernia happens more often?

1
Test 15 Answers

a) Male
b) Female
c) It is equal in both sexes

9. What is Pringle maneuver in liver trauma surgical procedures?


a) Liver parenchyma compression
b) Clamping of the hepatoduodenal ligament
c) Embolization of the common hepatic artery
d) Covering the bleeding surface with parts of the great omentum

10. Which are the components of echinococcal cysts ?


a) External laminated membrane
b) Internal germinative membrane
c) Outer pericyst made by host cells
d) Abscess cavity
e) Non defined borders

11. What is the clinical manifestation of liver echinococcosis?


a) Asymptomatic for years
b) Discomfort and heaviness in the right hypochondriac area, hives, nausea
c) Complications of the disease
d) All of the above

12. Which is the most common localization of liver echinococcosis?


a) Quadrate lobe
b) Left lobe
c) Right lobe

13. How could upper GIT bleeding be diagnosed?


a) Nasogastral tube insertion
b) X-ray with water soluble contrast material intake
c) Digital rectal examination
d) Abdominal ultrasound examination
e) CT scan
f) Upper endoscopy

14. What does Forest IA refers to?


a) Early gastric carcinoma without lymph node metastasis
b) Mucosa I esophageal carcinoma without lymph node metastasis
c) Spurting haemorrhage from a peptic ulcer
d) Diverticulosis of the ascending colon
e) Crohn's disease of the terminal ileum

15. Which are the indications for a surgical treatment o f a bleeding ulcer?
a) Forest IA type of haemorrhage

2
Test 15 Answers

b) Recurrent bleeding despite the conservative treatment


c) Decrease of the Hb levels after 24h from the beginning of the bleeding despite the conservative) treatment
d) Continuous blood loss and a necessity of haemotransfusions of more than 3 units of blood for 12-24h
e) All of the above

16. Which are the clinical signs of upper GIT bleeding?


a) Melaena
b) Haemoptoe
c) Haemathemesis
d) Abdominal pain
e) Vomiting of haematin materials

17. Which are the layers of the appendiceal wall?


a) Serosa
b) Muscular layer
c) Mucosa and muscular layer
d) Mucosa, muscular layer and serosa

18. Where is the localization of the appendiceal lymph nodes?


a) In the mucosa
b) In the submucosa
c) In the muscular layer
d) In the subserosa

19. Which group age has the greatest number of lymph follicles?
a) Children
b) Middle age
c) Old age

20. Which of the following disease must be included in the differential diagnosis of periappendiceal infiltrate in old
patients?
a) lntussusception
b) Bowel tuberculosis
c) Actinomycosis
d) Carcinoma of the cecum

21. Which of the following conditions could cause paralytic intestinal obstruction?
a) Acute peritonitis
b) Lead poisoning
c) Pancreatic necrosis
d) Retroperitoneal hematoma
e) Mesenteric ischemia

22. Which of the following conditions could lead to quick dehydration?


a) Small intestine torsion

3
Test 15 Answers

b) Sigmoid colon torsion


c) lleocecal intussusception
d) Neoplastic colon obstruction
e) Small intestines paresis

23. What is the origin of the splashing sound?


a) Abdominal cavity exudate
b) Fluid and gas collections in the adductor bowel loops
c) Fluid and gas collections in abductor bowel loops
d) Fluid and gas collections in the abdominal cavity
e) Non of the above

24. What is the clinical manifestation of intestinal strangulation?


a) Constant abdominal pain
b) Single vomiting
c) Recurrent vomiting
d) Colic abdominal pain
e) Splashing sound

25. What does treatment of acute paraproctitis include?


a) Antibiotic therapy
b) Incision
c) Local application of warm compress
d) Conservative treatment with jodasept solution
e) Drainage

26. What is chronic paraproctitis?


a) Perianal abscess
b) Anal fissure
c) Anal fistula
d) Internal and external hemorrhoids
e) Gas-producing phlegmone

27. Which of the following symptoms are typical for chronic paraproctitis?
a) Oedema, hyperemia, pain, fluctuation
b)Fever
c) Stools, mucus and pus leakage at regular intervals through an skin orifice around the anus
d) Painful skin formation around the anus
e) Fibrotic cord between the skin orifice and the anal canal

28. According to the PARKS classification anal fistulas could be classified as:
a) Suprasphinteric
b) lntrasphincteric
c) Transanal
d) Transsphincteric e) Extrasphincteric f) Horseshoe-like g) Pelviorectal

4
Test 16 Answers

1. Golden standard for diagnosing breast cancer is:


a) Physical examination, mammography, true-cut biopsy
b) Physical examination, CT-scan, express histological examination
c) Physical examination, MRI of the mammary gland, fine needle biopsy
d) Mammography, excision, express histological examination

2. What does modified radical mastectomy include?


a) Removing the mammary gland, minor pectoral muscle and axillary lymph nodes
b) Removing the mammary gland, pectoral fascia and axillary lymph nodes
c) Mammary gland and axillary lymph nodes saving procedure
d) Skin and axillary lymph nodes saving procedure

3. What is the first method of choice as a surgical procedure in a 52-year-old female patient with a histological
result of a luminal A-type of invasive ductal carcinoma, located in the upper lateral quadrant of the right mammary
gland, 12mm in size?
a) Extended radical mastectomy
b) Mammary gland saving procedure and sentinel lymph node biopsy
c) Skin saving procedure and axillary lymph node dissection
d) None of the above

4. Which of the following statements about sentinel lymph node biopsy is true?
a) It is a golden standard for determine theaxillary lymph node status of a patient with breast cancer
b) A lymphotropic clouring agent or a radioisotope could be used
c) If the sentinel lymph node is negative it could be accepted that there are no axillary lymph node metastasis
d) All of the above

5. Where is the exact anatomical position of Spigelian hernia?


a) The crossing point of the semicircular and semilunar lines
b) Above the umbilicus
c) In the epigastric region
d) Perineum
e) Non of the above

6. What is the definition of Richter's type of hernial incarceration?


a) Incarceration of antimesenteric portion of the bowel wall
b) Retrograde incarceration
c) Incarceration of a Meckel diverticulum
d) Non of the above

7. What is the definition of Litre's type of hernia incarceration?


a) Incarceration of antimesenteric portion of the bowel wall
b) Retrograde incarceration
c) Incarceration of a Meckel diverticulum
d) Non of the above

1
Test 16 Answers

8. Where does the necrotic process in retrograde hernia incarceration take place?
a) Inside the abdominal cavity
b) Outside the abdominal cavity
c) All of the above
d) Non of the above

9. What is the treatment of liver echinococcosis?


a) Conservative
b) PAIR procedure (puncture-aspiration-injection-reaspiration)
c) Echinococcectomy
d) Segment liver resection
e) All of the above

10. What is liver echinococcosis?


a) Bacterial infection of the liver
b) Viral infection
c) Parasite infection
d) Hematogenous disease

11. What is the etiology cause of liver abscesses?


a) Bile ducts diseases
b) Pylephlebitis
c) Posttraumatic changes of the liver *
d) Direct spread of the infection
e) Radiation

12. What is the treatment of solitary and confluent liver abscesses?


a) External open drainage
b) Percutaneous drainage
c) Endoscopic drainage

13. What is the possible diagnosis of 43-year-old patient with sharp and sudden abdominal pain, tachycardia
(lOlbpm), RR 115/70mmHg, pale skin, sweetening, no vomiting and rigid abdominal wall upon palpation?
a) Bleeding ulcer
b) Perforated ulcer
c) Exacerbated ulcer disease
d) Acute pancreatitis
e) Acute cholecystitis

14. Which methods are appropriate for diagnosing perforated ulcer?


a) Native X-ray
b) Barium sulfate X-ray
c) Abdominal ultrasound
d) CT scan
e) Laparoscopy

2
Test 16 Answers

f) ERCP

15. Which of the following symptoms are suspicious for a malignant transformation of a stomach ulcer?
a) Permanent abdominal pain
b) Pain in the epigastrium 10 minutes after feeding
c) Anemia
d) Stomach anacidity

16. Which is the most common complication of duodenal ulcers located on the anterior wall of the duodenum?
a) Malignancy
b) Perforation
c) Haemorrhage
d) Penetration to the head of the pancreas and the hepatoduodenal ligament

17. What specific features does acute appendicitis in children have?


a) Anatomical
b) Clinic
c) Age
d) There are no specific features

18. Which are the typical clinical features of acute appendicitis in children?
a) Pain occurs primary in the ileocecal area
b)Fever
c) Vomiting as a constant symptom
d) Atypical clinical manifestation

19. Which are the typical clinical features in acute appendicitis in old patients?
a) Lower incidence, periappendiceal infiltrate or abscess formation, intestinal obstruction complaints
b) Low body resistance to infections, destructive types of appendicitis
c) Poor clinical manifestation due to the tissue atrophy in old patients
d) All of the above

20. Which are the typical clinical features in acute appendicitis during pregnancy?
a) Upper localization of the spontaneous and palpatory pain
b) Difficult palpation due to growing uterus
c) Pain in the first half of pregnancy could be mistaken for abortion, in the second half of pregnancy for premature
la bor; vomiting in the first half of pregnancy could be mistaken for hyperemesis, in the second half for pregnancy
nephropathy
d) All of the above

21. Which are the typical X-ray findings in intestinal obstruction?


a) Cloiber glasses
b) Free gas under the diaphragm domes
c) Kerckring folds
d) Wahl sign

3
Test 16 Answers

e) Tsege-Manteuffel sign

22. What does the therapeutic measures in large bowel intestinal obstruction include?
a) Spasmolytic medications application
b) Enemas
c) Correction of the electrolyte imbalance
d) Opiod analgesics application
e) Application of medications increasing bowel motility

23. What is the clinical manifestation of ileocolic intussusceptions?


a) Bloody stools
b) Scybala stools
c) Tumor in the right iliac fossa
d) Colic abdominal pain
e) Anal sphincter atonia

24. What diagnostic methods must be used in a patients suspicious for intestinal obstruction?
a) Celiac trunk angiography
b) Upper endoscopy
c) Abdominal auscultation
d) Native abdominal X-ray
e) Digital rectal examination

25. Which of the following methods could be used for anal fistula exploration?
a) Fistula probe
b) Colonoscopy
c) Fistulography *
d) lrigography
e) Air or colouring agent injection

26. What does treatment of anal fistulas include?


a) Ligation
b) Local antibiotic application
c) Rectal resection
d) Tissue glue application
e) Puncture and excision

27. What is necessary for diagnosing a patient with acute abd o men syndro me?
a) Patient's history
b) Physical examina t io n
c) Laboratory tests and precise interpretation of the results
d) Image studies
e) All of the above

28. Acute abdomen is a: a) Sympto m b) Syndrome

4
Test 17 Answers

1. Which of the following are absolute contraindications fo performing a mammary gland saving procedure?
a) Patients in 1st or llnd stage
b) Multicentric carcinoma
c) Multifocal carcinoma
d) Good response to neoadjuvant chemotherapy

2. What is the medical term for the multidisciplinary approach which combines a single stage radical mastectomy
and a mammoplasty for better esthetic results?
a) Neoadjuvant treatment
b) Multimodal approach
c) Oncoplastic approach
d) Mammary gladn saving method

3. Congenital pulmonary cysts are developing as a result of neonatal defects in:


a) Mediastinal tissues
b) Terminal bronchi
c) Trachea and larynx

4. What is the medical term for the congenital absence of lungs and major bronchi ?
a) Agenesia
b) Aplasia
c) Hypoplasia

5. Which element forms the superficial ring of the inguinal canal?


a) Internal oblique muscle aponeurosis
b) External oblique muscle aponeurosis
c) Rectus abd ominis muscle aponeurosis
d) Transverse abdominal muscle aponeurosis

6. Which element forms the deep ring of the inguinal canal?


a) Internal oblique muscle aponeurosis
b) External oblique muscle aponeurosis
c) Rectus abdominis muscle aponeurosis
d) Transverse abdominal muscle aponeurosis
e) Transverse fascia

7. What is the content of the inguinal canal in women?


a) Broad ligament of the uterus
b) Round ligament of the uterus
c) Spermatic cord
d) All of the above
e) No n of the a bove

8. What is the content of the inguinal canal in men?


a) Broad ligament of the ute rus

1
Test 17 Answers

b) Round ligament of the uterus


c) Spermatic cord
d) All of the above
e) Non of the above

9. What is the treatment of superficial liver abscesses?


a) Antibiotic therapy
b) Surgical treatment
c) Percutaneous drainage under ultrasound control

10. Which of the following statements a bout the structure of the Iiver abscess is true?
a) Cavity full of pus, surrounded by a pyogenic membrane, varies in size
b) It has 3 membranes - fibrotic, laminated and germinative
c) It has no borders

11. Primary liver cancer could be:


a) Cystic formation
b) Solitary mass
c) Multifocal or infiltrative mass

12. Which are the late symptoms of primary liver cancer?


a) Loss ofappetite
b) Asthenoadynamia
c) Liver enlargement
d) Haemorrhage

13. Which of the following statements a bout the compensated stage of pyloric stenosis is true?
a) Splashing sound
b) Morning vomiting
c) Remained barium sulfate in the stomach for more than 12 h
d) Hypovolemia
e) Weight loss
f) Non of the above

14. Which are the clinical signs of a perforated ulcer?


a) Positive Blumberg sign in the first few hours after the beginning of the disease
b) Recurrent vomiting
c) Rigid abdominal wall
d) Absent liver dullness upon percussion
e) Splashing sound

15. Which are the clinical signs of malignant transformation of a chronic stomach ulcer?
a) Loss of appetite
b) Permanent pain in the epigastrium
c) Achilia

2
Test 17 Answers

d) Anemia
e) Elevated ESR
f) All of the a hove

16. What surgical procedure must be performed in a patient who was operated 24h after the beginning of the
complaints, with a 0,5cm perforation of an ulcer located on the anterior surface of the duodenum, 21 of purulent
exudate and bile in the abdominal cavity?
a) Stomach resection
b) Truncal vagotomy and pyloroplasty
c) Ulcer suture and abdominal cavity drainage
d) Ulcer excision and selective proximal vagotomy
e) Selective proximal vagotomy and antrumectomy

17. What is the treatment of acute appendicitis during pregnancy?


a) Antibiotics and physiotherapy
b) Surgical if the conservative therapy has no effect
c) Surgical treatment is contraindicated
d) Emergency surgery despite the pregnancy

18. What is the reason for primary gangrenous appendicitis in old patients?
a) Low resistance of the body
b) Atrophic changes
c) Embolism/ Thrombosis of the appendiceal artery

19. Which are the constant symptoms of acute appendicitis in middle age?
a) Fever
b) Muscular defense
c) Positive Blumberg sign

20. How many clinical forms of chronic appendicitis are there?


a) Chronic residual
b) Chronic recidive
c) Primary chronic
d) All of the above

21. In which of the following cases surgical treatment is indicated in intestinal obstruction?
a) Presence of Kloiber glasses after conservative treatment
b) Increasing abdominal pain
c) Acute peritonitis signs
d) Hypovolemia
e) Hypokalemia

22. Which of the following is contraindicated in patient's preparation for surgical treatment of intestinal
obstruction?
a) Stomach content aspiration

3
Test 17 Answers

b) Infusion therapy
c) Spasmolytic therapy
d) Application of medications increasing the peristalsis
e) Enemas

23. What does the surgical treatment of intestinal obstruction due to cecum carcinoma include?
a) Right hemicolectomy and ileotransverse anastomosis
b) Right hemicolectomy and temporary enterostomy
c) Collateral ileotransverse anastomosis
d) Temporary transverse colostomy
e) Temporary ileostomy

24. Which of the following conditions is part of the strangulated intestinal obstruction?
a) Volvulus
b) Gallstone intestinal obstruction
c) Nodule formation
d) Tumor compression
e) Bowel loop compression in a hernia sac

25. How many stages of muscular defense are there according to Mineur?
a) 1
b) 2
c) 3
d)4
e)S

26. Which of the following diseases could cause acute abdomen syndrome?
a) Hollow organs perforation
b) Inflammation of abdominal organs
c) Abdominal organs traumatic injuries
d) Intestinal obstruction
e) Early and late postoperative complications
f) All of the a hove

27. Could acute gastritis cause acute abdomen syndrome?


a) Yes
b) No

28. Which of the following statements a bout laparoscopic surgery is true?


a) Short hospital stay
b) Pain reduction
c) Fast recovery
d) Low incidence of surgical site infections and hernias
e) All of the above

4
Test 18 Answers

Test 18

1. How often does pulmonary echinococcosis develop?


a) It is the second most common localization after liver echinococcosis
b) It is the most common type
c) More rare than spleen and brain echinococcosis

2. Pulmonary echinococcosis growth is:


a) The fastest
b) The slowest
c) Growth depends on the localization

3. Which is the most dangerous complication of pulmonary echinococcosis?


a) Perforation in a bronchus
b) Compression of surrounding organs in the chest cavity
c) Pleural perforation

4. What is the surgical approach in a combined pulmonary and liver echinococcosis?


a) Surgical treatment of the liver localization first
b) Surgical treatment of the pulmonary localization first even in one stage procedures
c) None of the above

5. How many types of ingunal hernias are there?


a) Direct
b) Indirect
c) Congenital
d) Acquired
e) All of the above

6. Which type of ingunal hernia passes through the whole distance of the ingunal canal?
a) Direct
b) Indirect
c) All of the above
d) Non of the above

7. Which type of ingunal hernia doesn't pass through the inguinal canal?
a) Direct
b) Indirect
c) All of the above
d) Non of the above

8. Which type of inguinal hernia could reach the scrotum?


a) Direct
b) Indirect
c) All of the above

1
Test 18 Answers

d) Non of the above

9. What is the major treatment of hepatocellular carcinoma?


a) Liver resection
b) Tumor ablation
c) Liver transplantation

10. What is the treatment of advanced hepatocellular carcinoma?


a) Chemoembolization
b) Radiotherapy
c) Biliodigestive by pass procedures

11. Surgical treatment of liver metastases is indicated in:


a) Treatment of the primary tumor
b) Absence of systemic and other intraabdominal metastases
c) Radical resection of the primary tumor
d) It doesn't depend on the general condition of the patient

12. What is the treatment of gallbladder carcinoma?


a) Biliodigestive by pass procedures
b) Cholecystectomy
c) Cholecystectomy and liver resection of the 4th and the 5th segment
d) Right lobectomy
e) Right hemihepatectomy

13. What is the diagnosis of a patient with complaints of weakness, dizziness, vomiting and black stools without
any abdominal pain?
a) Atypical ulcer perforation
b) Pancreas necrosis
c) Pyloric stenosis
d) Mesenterial thrombosis
e) GIT bleeding

14. Which of the following diagnostic methods are used in GIT bleeding?
a) Native X-ray of the abdomen
b) Laparoscopy
c) Gastroduodenoscopy
d) Complete blood count test

15. Which are the specifi c features of decompensa ted pylo ric stenosis?
a) Vomiting of old food
b) Muscular defe nse
c) Splashing sound
d) Remaining barium sulfate contrast in the stomach for more than 24h

2
Test 18 Answers

16. Which are the typical clinical signs of perforated stomach ulcer?
a) Sharp sudden pain
b) Muscular defense
c) Constant vomiting
d) Absence of liver dullness upon percussion
e) Colic pain in the epigastrium

17. Which of the following diagnostic methods are most important for chronic appendicitis?
a) Native X-ray
b) Contrast X-ray
c) Colonoscopy
d) Abdominal ultrasound examination

18. Which age group is most affected from chronic appendicitis?


a) Early childhood
b) Teenagers
c) Middle age
d) Old age

19. What is the treatment of chronic appendicitis?


a) Prophylactic appendectomy
b) Physiotherapy
c) Emergency surgery
d) Elective surgery after contrast X-ray image studies and proved diagnosis

20. In which of the following age groups mesenterial lymphadenitis is a common disease?
a) Early childhood
b) Middle age
c) In all ages

21. In which of the following conditions colic abdominal pain is typical?


a) Neoplastic intestinal obstruction
b) Small intestine pa re sis after surgery of the aorta
c) Small intestine volvulus
d) Acute pancreatitis
e) Cecocecal intussusceptions
f) Mesenteric thrombosis

22. Which of the following is not typical for small bowel obstruction?
a) Tsege-Manteuffel sign
b) Abd ominal asymmetry
c) Splashing sound
d) Recurrent vomiting
e) Colic abdominal pain

3
Test 18 Answers

23. Which are the clinical features of large bowel obstruction?


a) Early recurrent vomiting
b) Stop offlatulence and defecation
c) Wahl sign
d) Abdominal distension
e) Tsege-Manteuffel sign

24. Which of the following diagnostic methods is a method of choice in intestinal obstruction?
a) Native abdominal X-ray
b) Laparoscopy
c) lrigography
d) Abdominal ultrasound
e) Colonoscopy

25. Which of the following is a contraindication for laparoscopic surgery?


a) Pregnancy
b) Peritonitis
c) Metastatic cancer
d) Heart failure
e) Chronic obstructive pulmonary disease

26. Which are the disadvantages of laparoscopic surgery?


a) 2D image
b) Difficult reatroperitoneal exploration
c) High cost
d) Abse nee of tactility
e) All of the above

27. Which methods could be used for better visualization in laparoscopic surgery?
a) Change of the patient's position
b) Retraction of the surrounding tissues
c) Placement of additional ports
d) Special patient's positions
e) All of the above

28. How a pneumoperitoneum be created for a laparoscopic surgery?


a) By using a Veres needle
b) Direct placement of the first trocar
c) Open laparoscopy by making a small incision for placing the first trocar under eye-control

4
Test 19 Answers

Test 19

1. What are the major histological types of lung cancer?


a) Small cell carcinoma and giant cell carcinoma
b) Small cell carcinoma and non small cell carcinoma
c) Non small cell carcinoma and polymorphous type

2. For a precise treatment of patients with advanced lung cancer there must be:
a) Histological verification and staging of the disease
b) Image studies
c) Surgical treatment and pain-killer administration

3. Malignant tumors of the lungs could be divided into:


a) Primary and benign
b) Primary and metastatic
c) Metastatic and cystic

4. Which is the obligatory condition for surgical treatment of a non-small cell lung carcinoma?
a) Histological verification
b) Histological verification and staging
c) Image and laboratory tests, staging

5. Which disease must be included in the differential diagnosis of inguinal hernia?


a) Femoral hernia
b) Funicular hydrocele
c) Inguinal lymphadenitis
d) Cryptorchidism
e) All of the above

6. In which of the following types of inguinal hernias the transmission of the cough impulse could be palpated on
the tip of the finger?
a) Direct hernia
b) Indirect hernia
c) All of the above
d) Non of the above

7. In which of the following types of inguinal hernias the transmission of the cough impulse could be palpated o n
the lateral surface of the finger?
a) Direct hernia
b) Indirect hernia
c) All of the above
d) Non of the above

8. Which inguinal region is most commonly affected in congenital inguinal hernias?


a) Left side

1
Test 19 Answers

b) Right side
c) Both sides

9. Which is the most common histological type of bile duct carcinoma?


a) Adenocarcinoma
b) Cystadenocarcinoma
c) Medullar carcinoma

10. What is the medical term for hepatic confluence tumors?


a) Courvoisier tumors
b) Klatskin tumors
c) Kocher tumors

11. What is the clinical manifestation of bile duct carcinoma?


a) Painless obstructive jaundice
b) Fever
c) Pain in the back

12. Which are the features of bile duct cancer treatment?


a) Paliative biliodigestive by pass procedures
b) Percutaneous transhepatic biliary drainage insertion
c) Paliative stent insertion in the common bile duct

13. Which of the following diagnostic methods could be used in a patient with muscular defense, positive
Blumberg sign, sharp sudden pain in the epigastrium, absence of liver dullness upon pe rcussion, heart rate 60bpm?
a) Upper endoscopy
b) X-ray series
c) Laparoscopy
d) Angiography
e) Native X-ray picture

14. Which of the following complications of ulcer disease manifests with melaena and absence of epigastrium
pain?
a) Pyloric stenosis
b) Perforation
c) Malignancy
d) Acute haemorrhage
e) Penetration t o pancreas

15. Which of the following diagnostic me thods is most informative in upper GIT bleeding?
a) X-ray series
b) Native X-ray picture
c) Upper endoscopy
d) Laparoscopy
e) Angiography

2
Test 19 Answers

16. Which of the following factors determine the choice of surgical procedure to be performed in a patient with
perforated ulcer?
a) Presence of acute peritonitis
b) The beginning of the complaints
c) Surgeon's qualification
d) Age and general condition of the patient
e) Size of the ulcer

17. How many forms of mesenterial lyphadenitis are there?


a) Generalized
b) Local
c) Local and generalized

18. What is the clinical manifestation of mesenterial lymphadenitis?


a) Abdominal pain without muscular defense, fever to 40 degrees, leucocytosis above 20 000, presence of
herpes
b) Abdominal pain with muscular defense, sebfebrility, leucocytosis under 20 000
c) No abdominal pain, palpable abdominal lymph nodes, diarrhea, vomiting

19. What is the infection's spreading way in mesenterial lyphadenitis?


a) Hematogenous
b) Enterogenous
c) Lymphogenous
d) All of the above

20. What is the treatment of mesenterial lymphadenitis?


a) Appendectomy and antibiotics
b) Co nservative treatment with antibiotics
c) Removal of all the abdominal lymph nodes

21. What is the therapeutic approach to a 75-year-old patient with sigmoid colon carcinoma and late large bowel
obstruction?
a) Physical examination and surgical treatment
b) Conservative treatment
c) Infusion therapy and enemas
d) Preoperative preparation for 2-3h and Hartmann procedure
e) Emergency surgery and primary end-to-end anasto mosis

22. What is the most co mmon cause for large bowel o bst ruction?
a) Foreign body
b) Gallsto nes
c) Malignant tumors
d) Adhesio ns
e) Helminths

3
Test 19 Answers

23. Small intestine volvulus is a type of:


a) Neoplastic intestinal obstruction
b) Strangulated intestinal obstruction
c) Mixed intestinal obstruction - neoplastic and strangulated
d) Spastic intestinal obstruction
e) Dynamic intestinal obstruction

24. Which of the following clinical features are typical for small bowel obstruction?
a) Recurrent vomiting
b) Severe constant pain in the mesogastrium
c) Colic abdominal pain
d) Splashing sound
e) Positive Schiemann sign

2S. Which of the following statements about laparoscopy assisted surgical procedures is false?
a) Some of the stages of the procedure are laparoscopic, others are open but through a mini laparotomy (below 6
cm)
b) They are using the advantages of both laparoscopic and open procedures for an optimal result
c) They are conversion from laparoscopic to open surgical procedures
d) They are faster and cheaper than the laparoscopic procedures alone

26. Which of the following statements about pneumoperitoneum is false?


a) Carbon dioxide is an inert gas, non-resorbable and harmless to human body
b) Compression of the inferior cava vein worsens the heart preloading
c) Carbon dioxide could cause subcutaneous emphysema if it is insufflated in the subcutaneous tissues
d) Carbon dioxide could cause air embolism if it is insufflated in a blood vessel

27. Which are the co mplications of pneumoperitoneum creatio n by the Veres needle?
a) Hollow organ 's injury
b) Arterial or venous vessel injury
c) Air embolism
d) All of the above

28. In which of the following cases bile ducts lesions could be suspected after laparoscopic cholecystectomy?
a) Postoperative jaundice
b) Bile discharge through the subhepatic tube drainage
c) All of the above

4
Test 20 Answers

Test 20

1. Non complicated lung abscess is:


a) Local purulent infection of the lung parenchyma
b) Diffuse purulent infection
c) Pleural empyema

2. Which are the typical features of lung gangrene?


a) There is no pyogenic membrane
b) Spreading of the process to the adjacent lobe or the whole lung
c) There are purulent- necrotic sequesters in the centre of the affected lung
d) All of the above

3. How could lung abscesses be classified according to their etiology?


a) Pneumogene
b) Aspiration
c) Traumatic
d) Obturative
e) Haematogenous
f) Lymphogenous
g) Thromboembolic
h) All of the above

4. Which are the possible complications of lung abscess?


a) Pyothorax
b) Haemoptoe
c) Bronchiectasis
d) Septicopyemic metastasis
e) Chronification
f) Malignancy

5. Which type of inguinal hernia is more rare?


a) Indirect
b) Direct

6. Which type of inguinal hernia is more often bilateral?


a) Indirect
b) Direct

7. In which type of inguinal hernia patients could have dysuria?


a) Indirect
b) Direct
c) Su pravesical

8. What is the treatment for inguinal hernia?


a) Surgical
b) Conservative
c) All of th e above

9. What is Courvoisier sign?

1
Test 20 Answers

a) Enlarged painful gallbladder in a patient with jaundice


b) Enlarged painless gallbladder in a patient with jaundice
c) Enlarged painless gallbladder in a patient with no jaundice
d) It is positive in pancreatic head carcinoma
e) It is positive in cholangitis

10. In which of the following cases contrast CT-scan is contraindicated in patients with acute pancreatitis?
a) Absolutely contraindicated due to the worsening of the necrotizing process and developing of acute kidney failure
b) Contraindicated only in patients with kidney insufficiency
c) It is not indicated for diagnosing acute pancreatitis

11. What is the possible diagnosis of a 72-year-old patient with severe abdominal pain, numerous vomitings, epigast rium pain
upon palpation with no muscular defense, increased amylase levels, leucocytosis and borderline levels of MB-CPK?
a) Acute myocardial infarction
b) Acute pancreatitis
c) Bowel obstruction
d) Stomach ulcer perforation
e) Gastroduodenitis

12. Therapeutic approach to a patient with acute pancreatitis includes everything mentioned except:
a) Starving
b) Good hydration
c) Antibiotic therapy
d) Emergency surgery

13. Which of the following symptoms are typical fo duodenal ulcer perforation?
a) Vomiting of haematin materials
b) Absence of liver dullness upon percussion
c) Muscular defense
d) Sudden sharp pain
e) Splashing sound

14. Native abdominal X-ray picture in a patient with sudden abdominal pain shows subdiaphragmatic free gas. What is the
diagnosis?
a) Acute pancreatitis
cb) Acute cholecystitis
d) Stomach or duodenal ulcer pe rforation
e) Intestinal obstruction
f) Acute appendicitis

15. Which of the following complications of duodenal ulcer is false?


a) Penetration to the hepatoduodenal ligament
b) Malignant transformation
c) Retroperitoneal perforation
d) Acute GIT bleeding
e) Pyloric stenosis

16. Which of the following pathophysiological diso rders are typical for decompensated pylo ric st enosis?
a) Hypervolemia
b) Anemia

2
Test 20 Answers

c) Metabolic alkalosis
d) Hypovolemia
e) Respiratory acidosis
f) Hypokalemia

17. What is the incidence of Meckel's diverticulum?


a)20%
b) 2%
c) 50%
d)100%

18. Meckel's diverticulum is a remnant of:


a) Placenta
b) Omphaloenteric duct
c) Thyroglossal duct
d) All of the above

19. What is the localization of Meckel's diverticulum?


a) Esophageal
b) Small bowel
c) Colon
d) Duodenal

20. How far from the ileocecal valve could Meckel's diverticulum be found?
a) More than 200cm
b) Less than 200cm

21. Which type of intestinal obstruction leads to quick intestinal necrosis?


a) External compression
b) Small bowel volvulus
c) Gallstone intestinal obstruction
d) Nodule formation
e) Bowel loop incarceration

22. Which of the following diseases could lead to a colic abdominal pain after physical loading in a 45-year-old, double vomiting,
abdominal distension and stop of flatulence?
a) Stomach ulcer perforation
b) Acute intestinal obstruction
c) Acute pancreatitis
d) Acute appendicitis
e) Acute cholecystitis

23. Which is the most common localization of large bowel obstruction?


a) Cecum
b) Hepatic flexure
c) Spleen flexure
d) Middle third of the transverse colon
e) Sigmoid colon

24. Which of the following diagnostic methods could be used in acute intestinal obstruction?

3
Test 20 Answers

a) Gastroduodenoscopy
b) Gastroscopy
c) Native abdominal X-ray
d) lrigography
e) All of the above

25. What diagnostic methods could be used for diagnosing a bile ducts lesion after laparoscopic cholecystectomy?
a) Laboratory tests
b) ERCP
c) MRI cholangiography
d)PTC
e) Abdominal ultrasound
f) All of the above

26. What is necessary for performing a laparoscopic surgical procedure?


a) Qualified team
b) Laparoscopic technical devices
c) All of the above
d) Non of the above

27. What gas is used for creating pneumoperitoneum for laparoscopic surgery?
a) Oxygen
b) Hydrogen
c) Carbon
d) Carbon dioxide
e) Nitrogen dioxide

28. What BMI is recommended for a laparoscopic surgery?


a) >18
b) <35
c) Above 35
d) All of the above
e) None of the above

4
Test 21 Answers

Test 21

1. What does the treatment of the lung abscess include?


a) I.V. infusions of antibiotics
b) Local application of antibiotics
c) Surgical treatment

2. What does the treatment of the lung gangrene include?


a) Drainage
b) Pulmonectomy
c) Segment lung resection

3. What is the most common localization of lung abscesses?


a) Right lower lobe
b) Right apical lobe
c) Left lobe

4. Which of the following statements are true for bronchiectasis?


a) Left lung is most commonly affected
b) It is a nonspecific chronic inflammatory-destructive disease
c) It affects totally the bronchi wall and the surrounding lung parenchyma, including the blood and lymph
vessels
d) Reversible dilation of the bronchi

5. Which of the following surgical procedures reinforce the anterior wall of the inguinal cana l?
a) Bassini technique
b) Martinov technique
c) Gerard technique
d) Band C
e) All of the above

6. Which of the following surgical procedures reinforce the posterior wall of the inguinal canal?
a) Bassini technique
b) Martinov technique
c) Gerard technique
d) All of the above

7. Which of the foll owing surgical procedures reinforce both the anterior and posterior wall of the inguinal canal?
a) Postemsky technique
b) Bassini technique
c) Gerard technique
d) All of the above

8. Which of the following types of hernioplasty includes using a mesh?


a) Postemsky technique

1
Test 21 Answers

b) Bassini technique
c) Gerard technique
d) Lichtenstein technique
e) All of the above

9. What is the cause of collapse and shock in acute pancreatitis?


a) Cholemia and compression of the terminal part of the common bile duct
b) Enzymatic toxemia
c) Biliary hypertension
d) Dynamic bowel obstruction

10. Which of the following diagnostic methods is the most informative one for necrotizing pancreatitis?
a) Laparoscopy
b) Ultrasound examination
c) Upper endoscopy
d) Urine analysis of the amylase levels
e) Pancreatic enzyme levels in the blood

11. What is the diagnosis of a patient who is undergoing exlplorative laparotomy during which there is a big
amount of exudate in the abdominal cavity and numerous necrotic lesions?
a) Tuberculous peritonitis
b) Steatonecrosis of the pancreas
c) Crohn's disease
d) Non occlusive thrombosis of the small bowels
e) All the answers are wrong

12. Which are the etiology risk factors for developing chronic pancreatitis?
a) Chronic alcohol abuse
b) Cholelithiasis
c) Malnutrition
d) All of the above

13. Which of the following types of ulcer perforation are atypical?


a) Perforation to the free abdominal cavity
b) Penetration to the lesser omentum
c) Perforation to the omental bursa
d) Perforation to the retroperitoneal space

14. Which of the following statements a bout pyloric stenosis is not true?
a) Contrast material X-ray is informative for the diagnosis
b) Upper endoscopy is informative for the diagnosis
c) Patients have hypokalemia
d) Clinical manifestation of the disease is identical to that of an advanced distal stomach carcinoma

15. What does Borchardt triade include?

2
Test 21 Answers

a) Mechanical jaundice, fever, pain in the right hypochondriac area


b) Pulselessness, pale col or and pain in the lower extremities
c) Sharp abdominal pain, vomiting, impossibility for a nasogastric tube insertion
d) Absence offlatulence, abdominal distension, splashing sound upon auscultation
e) Severe abdominal pain, pneumoperitoneum, muscular defense

16. In which disease there is a positive Bergman sign?


a) Acute pancreatitis
b) Ulcer perforation
c) Bleeding ulcer
d) Acute cholecytitis

17. What is the clinical manifestation of Meckel' s diverticulum?


a) Diverticulitis
b) Intussusception
c) Strangulation
d) Hemorrhage
e) Asymptomatic
f) All of the a hove

18. What is the characteristic of Meckel's diverticulum bleeding?


a) Rectorrhagia
b) Melaena
c) Raspberry jelly stools

19. In which of the following cases Meckel's diverticulum must not be removed?
a) Acute peritonitis due to another disease
b) Hemoperitoneum for example due to spleen rupture
c) If the patient is a newborn
d) If the surgical team is not qualified

20. What kind of surgical procedure must be performed for a Meckel's diverticulum removal?
a) Appendectomy
b) Wedge resectio n
c) Lesion resection
d) Segmental resection and end-to-end anastomosis

21. What is the clinical manifestation o f strangulated intestinal obstruction?


Contant abdominal pain
a) Single vomiting
b) Recurrent vomiting
c) Colic abdominal pain
d) Splashing sound

22. What surgical procedure is a method of choice in sigmoid colon volvulus?

3
Test 21 Answers

a) Hartmann resection
b) Sigmoid colon plication
c) Devolvation
d) Resection and primary anastomosis
e) Suture

23. What is the treatment of dynamic intestinal obstruction?


a) Conservative
b) Surgical
c) Surgical if the conservative one is not effective
d) Laparoscopic ileostomy
e) Large bowel tube insertion

24. Which of the following types of intestinal obstruction does not lead to mesenteric blood supply disorders?
a) Volvulus
b) Neoplastic obstruction
c) Nodule formation
d) Intussusceptions
e) All of the above

25. Is a patient in shock suitable for a laparoscopic surgery?


a) No
b) Yes
c) Depending on the patient's condition

26. What kind of surgical treatment is better for a patient requiring a pelvic surgical procedure if there is a
qualified team and good technical suppliances?
a) Open surgery
b) Laparoscopic surgery

27. Neoplastic intestinal obstruction is:


a) Mechanical type
b) Dynamic type

28. How could a Meckel' s diverticulum be diagnosed?


a) Abdo minal ultrasound examination
b) Colonoscopy and irigography
c) Explorative laparotomy

You might also like